Win up to 100% Scholarship

Register Now

Science & Technology

 

2022

 

Question 1

“Biorock technology” is talked about in which one of the following situations?

(a) Restoration of damaged coral reefs

(b) Development of building materials using plant residues

(c) Identification of areas for exploration/extraction of shale gas

(d) Providing salt licks for wild animals in forests/protected areas

Ans: a

Sub-Theme: Coral Reef restoration

Option (a) is correct: Recently, The Zoological Survey of India (ZSI), with help from Gujarat’s forest department, is attempting for the first time a process to restore coral reefs using biorock or mineral accretion technology in the Gulf of Kachchh.

  • Biorock is the term used to describe the material created by the electro- accumulation of minerals dissolved in saltwater on steel structures that are lowered onto the seafloor and connected to an energy source, in this case solar panels that float on the surface.

 

Question 2

Consider the following statements :

DNA Barcoding can be a tool to:

  1. assess the age of a plant or animal.

  2. distinguish among species that look alike.

  3. identify undesirable animal or plant materials in processed foods.

Which of the statements given above is/are correct?

(a) 1 only

(b) 3 only

(c) 1 and 2

(d) 2 and 3

Ans: d

Sub-Theme: DNA

Statement 1 is incorrect: Scientists use a radioactive carbon isotope called carbon-14 as their clock to determine the age of plant and animal remains from the more recent past. DNA barcoding is a taxonomic technique that identifies an organism as belonging to a specific species by using a short genetic marker in its DNA. An individual DNA sequence is compared to a reference library of similar DNA sections to accomplish this (called sequences). Statement 2 is correct: DNA Barcoding helps in the identification of species even from minute amounts of tissue which can help tackle problems like illegal trade of endangered species. DNA barcodes can effectively control pest attack, identifying disease vectors properly.

Statement 3 is correct: Using DNA barcoding, which is widely used in fish species, is a viable method for authenticating the processed food. Using DNA barcoding, natural resource managers can monitor illegal trade of products made of natural resources like hardwood trees. DNA barcoding can be used to identify the important medicinal plant.

 

Question 3

Consider the following statements in respect of probiotics:

  1. Probiotics are made of both bacteria and yeast.
  2. The organisms in probiotics are found in food we ingest but they do not naturally occur in our gut.
  3. Probiotics help in the digestion of milk sugars.

Which of the statements given above is/are correct?

(a) 1 only

(b) 2 only

(c) 1 and 3

(d) 2 and 3

Ans: c

Statement 1 is correct: Probiotics are live bacteria and yeasts that are believed to be helpful in preventing a number of diseases. They are also referred to as “good bacteria” and are typically eaten as supplements or in yogurt, also found in other fermented foods

Statement 2 is incorrect: Some bacteria aid in food digestion, eliminate disease-causing cells, or create vitamins. Numerous probiotic product bacteria are identical to or closely related to those found in human bodies naturally.
Statement 3 is correct: A well-known food source of probiotics is yoghurt. The enzyme ß-D-galactosidase is present in some strains of bacteria in yoghurt, aids in the breakdown of lactose in dairy products into the sugars glucose and galactose. Thus it also helps in the digestion of milk sugars.

 

Question 4

Which one of the following statements best describes the role of B cells and T cells in the human body?

(a) They protect the body from environmental allergens.

(b) They alleviate the body’s pain and inflammation.

(c) They act as immune suppressants in the body.

(d) They protect the body from the diseases caused by pathogens.

Ans: d 

Sub-Theme: Basic Science

Option (d) is correct: Lymphocytes are primarily classified into B cells and T cells. B cells function by producing antibodies that are released into the body’s fluids, which then attack antigens present in the bloodstream.

  • Acquired immunity is pathogen specific, it is characterized by memory.
  • This means when our body encounters a pathogen for the first time it produces a response called primary response which is of low intensity.
  • Subsequent encounters with the same pathogen elicits a highly intensified secondary or anamnestic response.
  • This type of immune responses are carried out with the help of two special types of lymphocytes present in our blood, i.e B-lymphocytes and T-lymphocytes.
  • The B-lymphocytes produce an army of proteins in response to pathogens into our blood to fight with them.
  • These proteins are called antibodies.

 

Question 5

With reference to “Software as a Service (SaaS)”, consider the following statements:

  1. SaaS buyers can customise the user interface and can change data fields.

  2. SaaS users can access their data through their mobile devices.

  3. Outlook, Hotmail and Yahoo Mail are form of SaaS

Which of the statements given above are correct?

(a) 1 and 2 only

(b) 2 and 3 only

(c) 1 and 3 only

(d) 1, 2 and 3

Ans: d

Sub-Theme: Information Technology

Software as a Service (SaaS)

  • Software as a Service means software which is offered by a third party as a service, which means that hosting and maintenance are taken care of by the third
  • Previously if an organisation wanted to use a software, it had to be deployed on local systems and then maintained appropriately for usage. But in SAAS, software can be directly accessed typically using thin clients like browsers.
  • Customers can adjust the user interface (UI) to alter the program’s appearance and feel as well as particular components, like data fields, to change the data that is
  • SaaS enables online programme access for any user. The user is not required to install the programme on their computer. Users can therefore access data via their mobile
  • SaaS enables users to access the software from several places via a web With remote desktop software, he or she may work from home.
  • SaaS includes services like Outlook, Hotmail, and Yahoo Mail.

Statement 1 is correct: Web-based software is adaptable enough to be customised for both particular corporate needs and individual users. Customers can adjust the user interface (UI) to alter the program’s appearance and feel as well as particular components, like data fields, to change the data that is displayed.

Statement 2 is correct: SaaS enables online programme access for any user. The user is not required to install the programme on their computer. Users can therefore access data via their mobile devices.

Statement 3 is correct: SaaS includes services like Outlook, Hotmail, and Yahoo Mail. In order to utilise these services, a user must log into their account online, frequently using a computer browser.

 

Question 6.

With reference to Web 3.0, consider the following statements:

  1. Web 3.0 technology enables people to control their own data.

  2. In Web 3.0 world, there can be blockchain based social networks.

  3. Web 3.0 is operated by users collectively rather than a corporation.

Which of the statements given above are correct?

(a) 1 and 2 only

(b) 2 and 3 only

(c) 1 and 3 only

(d) 1, 2 and 3

Ans: d

Sub-Theme: Web 3.0 Technology

Web3

  • In a Web3 world, people control their own data and bounce around from social media to email to shopping using a single personalised account, creating a public record on the block chain of all of that
  • Web 3.0 will be permissionless and democratic. Thus people can maintain blockchain based social networks in Web 3.0.
  • Web 3.0 is a decentralized internet to be run on blockchain technology.

World Wide Web

  • World Wide Web, also known as a Web, is a collection of websites or web pages stored in web servers and connected to local computers through the internet.
  • These websites contain text pages, digital images, audios, videos, Users can access the content of these sites from any part of the world over the internet using their devices such as computers, laptops, cell phones, etc.

Statement 1 is correct: In a Web3 world, people control their own data.

Statement 2 is correct: Web 3.0 will be permissionless and democratic. Thus people can maintain blockchain based social networks in Web 3.0.

Statement 3 is correct: In Web 3.0 all the value that’s created can be shared amongst more people, rather than just the owners, investors and employees. Thus it is operated by users collectively rather than by corporations.

 

Question 7

Which one of the following is the context in which the term “qubit” is mentioned?

(a) Cloud Services

(b) Quantum Computing

(c) Visible Light Communication Technologies

(d) Wireless Communication Technologies

Ans: b

Sub-Theme: Quantum Computing

The Correct answer is Option (b): Quantum computers use quantum bits or Qubits, which can be both 0 and 1 at the same time. Quantum Computing

  • Quantum computers are machines that use the properties of quantum physics to store data and perform computations.
  • A classical computer performs operations using classical bits, which can be either 0 or 1.
  • Quantum computer uses quantum bits or Qubits, which can be both 0 and 1 at the same time.
    • Qubits (or quantum bits) are the units of computation in quantum computers. They take advantage of quantum mechanics characteristics, which regulate how matter behaves at the atomic level.

The laws of quantum physics are used to achieve functioning of quantum computing:

  1. Superimposition: Each quantum bit (basic unit of information in a quantum computer) can represent both a 1 and a 0 at the same time, which is known as
  2. Quantum entanglement: Subatomic particles become “entangled” (connected) in quantum entanglement, which means that any change in one upsets the other, even though they are at opposite ends of the universe.
  • Major advantages: Faster, Accurate, & Energy efficient.
    • Sycamore: it is Google’s Quantum Computer, which recently claimed Quantum Supremacy.
    • Quantum Supremacy: refers to quantum computers being able to solve a problem that a classical computer

 

Question 8

Consider the following communication technologies:

  1. Closed-circuit Television

  2. Radio Frequency Identification

  3. Wireless Local Area Network

Which of the above are considered Short-Range devices/technologies?

(a) 1 and 2 only

(b) 2 and 3 only

(c) 1 and 3 only

(d) 1, 2 and 3

Ans: d

Sub-Theme: Short Range Devices

The Correct answer is Option (d): All the given technologies are Short Range Technologies.

Short Range Device/Technologies

  • The term “short-range radio device” refers to radio transmitters that offer either unidirectional or bidirectional communication and have a low potential to interfere with other radio
  • Such devices are allowed to operate without hindrance and without being protected.

The term “Short Range Device” refers to a wide range of wireless devices, such as different types of:

  1. Access control (including door and gate openers)
  2. Alarms and movement detectors
  3. Closed-circuit television (CCTV)
  4. Cordless audio devices, including wireless microphones
  5. Industrial control
  6. Local Area Networks
  7. Medical implants
  8. Metering devices
  9. Remote control
  10. Radio frequency identification (RFID)
  11. Road Transport Telematics
NOTE: Recently, Google Pay has launched a new feature in India, ‘Tap to pay for UPI’, in collaboration with Pine Labs. The feature makes use of Near Field Communication (NFC) technology, which is a short-range wireless connectivity technology.

 

Question 9

Consider the following statements:

  1. Other than those made by humans, nanoparticles do not exist in nature.

  2. Nanoparticles of some metallic oxides are used in the manufacture of some cosmetics.

  3. Nanoparticles of some commercial products which enter the environment are unsafe for

Which of the above statements is/are correct?

(a) 1 only

(b) 3 only

(c) 1 and 2 only

(d) 2 and 3

Ans: d

Sub-Theme: Nanoparticles

Nanoparticles

  • Particles with particle size less than 100 nm are called nanoparticles. Nanoparticles are particles with dimensions comparable to 1/109 of a metre.
  • Types of nanoparticles:
  • Carbon Based: It includes Graphene and other carbon based products.
  • Metal based: The main component of these nanomaterials are metals. They include Nano silver and metal oxides.
  • Natural NPs: sourced from forest fires, volcanic eruptions, weathering, dust storms etc. Eg: iron oxides/sulfides, silver, and
  • Anthropogenic NPs: unknowingly or purposely released in the environment during various industrial and mechanical
  • Cosmetics: In order to provide an adequate level of sun protection, titanium dioxide and zinc oxide are frequently added to cosmetics among the metal and metal oxide nanoparticles (NPs) that may be present.
  • Unsafe to Humans:
    • There are various methods that may be used to explain how humans and the environment are exposed to nanoparticles.
    • Workers including engineers, scientists, and technicians are mostly exposed at work while creating goods using nanomaterials on a commercial scale and at research size.
    • This exposure primarily comes from handling raw materials while performing reactions using the apparatus.
    • The second stage involves exposing customers to such nanomaterials through usage and application, which may have hazardous and detrimental impacts.
  • Statement 1 is incorrect: Natural NPs are sourced from forest fires, volcanic eruptions, weathering, dust storms etc. Eg: iron oxides/ sulphides, silver, and gold.
  • Statement 2 is correct: In order to provide an adequate level of sun protection, titanium dioxide and zinc oxide are frequently added to cosmetics among the metal and metal oxide nanoparticles (NPs) that may be present.
  • Statement 3 is correct: Workers including engineers, scientists, and technicians are mostly exposed at work while creating goods using nanomaterials, while on the other hand customers get exposed to such nanomaterials through usage and application, which may have hazardous and detrimental impacts.

 

Question 10

If a major solar storm (solar flare) reaches the Earth, which of the following are the possible effects on the Earth?

  1. GPS and navigation systems could fail.

  2. Tsunamis could occur at equatorial regions.

  3. Power grids could be damaged.

  4. Intense auroras could occur over much of the Earth

  5. Forest fires could take place over much of the planet.

  6. Orbits of the satellites could be disturbed.

  7. Shortwave radio communication of the aircraft flying over polar regions could be interrupted.

Select the correct answer using the code given below:

(a) 1, 2, 4 and 5 only

(b) 2, 3, 5, 6 and 7 only

(c) 1, 3, 4, 6 and 7 only

(d) 1, 2, 3, 4, 5, 6 and 7

Ans: c

Sub-Theme: Space Activity

SOLAR FLARE

  • NASA’s Solar Dynamics Observatory (SDO) studies how the Sun creates solar activity and drives space weather.
  • Solar flares are a sudden explosion of energy caused by reorganizing of magnetic field lines near sunspots.
  • Solar magnetic cycle that works in deep interior of Sun creates regions that rise to surface and appear like dark spots, known as sunspots.
  • They appear dark because they are cooler than other parts of Sun’s surface.
  • In a solar flare, energy stored in sun’s magnetic structures is converted into light and heat energy.
  • This causes emission of high energy x-ray radiation and highly accelerated charged particles to leave the sun’s surface.
  • Sometimes solar flares also cause hot plasma to be ejected from Sun, causing a solar storm, and this is called Coronal Mass Ejection (CME).
  • Energy, radiation and high energy particles emitted by solar flares can trigger intense lights in the sky, called Auroras.
  • These solar explosions propel bursts of particles and electromagnetic fluctuations into Earth’s atmosphere that could destroy transformers in power systems. Particles from a CME can potentially collide with a satellite’s critical electronics, causing the satellite’s systems to malfunction.

Expected high-frequency communication blackouts, satellite  anomalies,  GPS scintillations, airline communication impacts.

  • Statements 1 and 7 are correct: These solar explosions propel bursts of particles and electromagnetic fluctuations into Earth’s atmosphere that could lead to the malfunction of satellites, which might impact high-frequency communication blackouts, satellite anomalies, GPS scintillations, airline communication.
  • Statement 2 is incorrect: Tsunamis occur due to seismic activity.
  • Statement 3 is correct: The solar explosions propel bursts of particles and electromagnetic fluctuations into Earth’s atmosphere that could destroy transformers in power systems Statement 4 is correct: Energy, radiation and high energy particles emitted by solar flares can trigger intense lights in the sky, called Auroras. Statement 5 is incorrect: Flares are electromagnetic radiation bursts rather than actual fires. They won’t start forest fires as a result.
  • Statement 6 is correct: Particles from a CME can potentially collide with a satellite’s critical electronics, causing the satellite’s systems to malfunction.

 

Question 11

Which one of the following statements best reflects the idea behind the “Fractional Orbital Bombardment System” often talked about in the media?

(a) A hypersonic missile is launched into space to counter the asteroid approaching the Earth and explode it in space.

(b) A spacecraft lands on another planet after making several orbital motions.

(c) A missile is put into a stable orbit around the Earth and deorbits over a target on the Earth.

(d) A spacecraft moves along a comet with the same speed and places a probe on its surface

Ans: c

Sub-Theme: Defence Infrastructure

The Correct answer is Option (c): Fractional Orbital Bombardment System

fractional orbital bombardment system

  • A Fractional  Orbital  Bombardment
  • System (FOBS) is a warhead delivery system that uses a low earth orbit towards its target destination. Just before reaching the target, it de-orbits through a retrograde engine burn.
  • The Soviet Union first developed FOBS as a nuclear-weapons delivery system in the
  • In August 2021, the People’s Republic of China tested a weapon that combined a FOBS with a hypersonic glide vehicle.
  • It had no range limit, its flight path would not reveal the target location, and warheads could be directed to North America over the South Pole, evading detection by NORAD’s north-facing early warning systems.
  • The maximum altitude would be around 150 km.
  • Energetically, this would require a launch vehicle powerful enough to be capable of putting the weapon ‘into orbit’.
  • However the orbit is only a fraction of a full orbit, not sustained, and so there would be much less need to control a precise orbit, or to maintain it long term.

 

Question 12

Consider the following statements:

  1. Biofilms can form on medical implants within human tissues.

  2. Biofilms can form on food and food processing surfaces.

  3. Biofilms can exhibit antibiotic resistance.

Which of the statements given above are correct?

(a) 1 and 2 only

(b) 2 and 3 only

(c) 1 and 3 only

(d) 1, 2 and 3

Ans: d

Sub-Theme: Microorganisms

  • Statement 1 and 2 are correct: Biofilms may form on a wide variety of surfaces, including living tissues, food and food processing surfaces, indwelling medical devices (devices in the body like catheters, heart valves), human and animal tissue, industrial or potable water system piping, or natural aquatic systems.
  • Statement 3 is correct: As Biofilms attach to each other and to the surfaces, they are capable of acting as barriers to antibiotics.

Biofilms

  • A biofilm is a collection of microbial cells that are encased in a polysaccharide-based matrix and are permanently attached to a surface (i.e., cannot be removed by gentle rinsing).
  • Van Leeuwenhoek, using his simple microscopes, first observed microorganisms on tooth surfaces and can be credited with the discovery of microbial biofilms.
  • Microorganisms that form biofilms include bacteria, fungi, and protists.
  • Noncellular materials such as mineral crystals, corrosion particles, clay or silt particles, or blood components, depending on the environment in which the biofilm has developed, may also be found in the biofilm

 

2021

Question 1

Bollgard I and Bollgard II technologies are mentioned in the context of

(a) Clonal propagation of crop plants

(b) Developing genetically modified crop plants

(c) Production of plant growth substances

(d) Production of biofertilizers

Ans: b

Sub-Theme: Genetics/Biotechnology/ GM Crops

Option (b) is correct:

  • Bollgard I (single-gene technology) is India’s first biotech crop technology (Genetically modified crop) approved for commercialization in India in 2002, followed by Bollgard II – (double-gene technology) in mid-2006.
  • Bollgard cotton has an insecticidal protein froma naturally occurring soil microorganism called Bacillus thuringiensis and offers built-in protection for cotton against destructive American Bollworm Heliothis Armigera infestations (Bt).
  • The enhanced double-gene Cry1Ac and Cry2Ab technology found in Bollgard II technology protects against bollworms and Spodoptera caterpillars, improving boll retention, maximising yield, using fewer pesticides, and preventing insect
  • NOTE: Biotechnology is an important part of Science and tech syllabus reading class 12 ncert and daily newspaper might come handy while answering such questions. Also, this issue was very much in the news, hence, UPSC asked the question.

 

Question 2

In the context of hereditary diseases, consider the following statements:

  1. Passing on mitochondrial diseases from parent to child can be prevented by mitochondrial replacement therapy either before or after in vitro fertilization of the egg.

  2. A child inherits mitochondrial diseases entirely from mother and not from father.

Which of the statements given above is/are correct?

(a) 1 only

(b) 2 only

(c) Both 1 and 2

(d) Neither 1 nor 2

Ans: c

Sub-Theme: Disease transmission

  • Mitochondrial replacement therapy (MRT) also known as Mitochondrial Gene Therapy or Mitochondrial Donation is a new form of reproductive in-vitro fertilization (IVF) which works on the principle of replacing a women’s abnormal mitochondrial DNA (mt-DNA) with the donor’s healthy one.
    • Through the in vitro fertilization technique (IVF), the egg is then fertilized with the partner’s sperm. Thus the embryo remains free from any such defects.
  • Statement 1 is correct: The mitochondrial donation can be performed either prior to or shortly after fertilization. In both cases, this is done before the fertilized egg becomes an embryo.
  • Statement 2 is correct: Mitochondria is the only organelle in addition to the nucleus which contains DNA and genes. The sperm contains a very low number of mitochondria and mitochondrial genes. So, in the offspring, the mitochondrial genes are inherited from the mother. Thus, a father with a mitochondrial gene defect cannot transmit the disease to his offspring.

 

Question 3

The term ‘ACE2’ is talked about in the context of:

(a) Genes introduced in the genetically modified plants

(b) The development of India’s own satellite navigation system

(c) Radio collars for wildlife tracking

(d) The spread of viral diseases

Ans: d

Sub-Theme: Disease transmission

Option (d) is correct: Angiotensin Converting Enzyme 2 (ACE2) has been discovered to be the SARS-CoV-2 virus’s receptor.

  • Angiotensin Converting Enzyme 2 (ACE2) has been discovered to be the SARS-CoV-2 virus’s receptor.
  • The coronavirus can enter a variety of human cells through this particular protein and spread infection.
  • Numerous cell types and tissues, such as the kidneys, liver, gastrointestinal system, blood arteries, lungs, and heart, all contain ACE2. It is found in epithelial cells, which line several tissues and build defences.
  • Across this epithelial lining in the lung, oxygen and carbon dioxide are exchanged between the lungs and blood vessels. The epithelium in the nose, mouth, and lungs contains ACE2.

 

Question 4

Consider the following statements:

  1. Adenoviruses have single stranded DNA genomes whereas retroviruses have double-stranded DNA genomes.

  2. Common cold is sometimes caused by an adenovirus whereas AIDS is caused by a an adenovirus whereas AIDS is caused by a retrovirus.

Which of the statements given above is/are correct?

(a) 1 only

(b) 2 only

(c) Both 1 and 2

(d) Neither 1 nor 2

Ans: b

Sub-Theme: Viral diseases

  • Statement 1 is incorrect: The icosahedral viruses known as adenoviruses (AdVs) have double-stranded DNA (dsDNA) genomes whereas retroviruses use RNA(Single stranded) as its genetic material.
    • Adenoviruses are a family of widespread viruses that affect your brain system, intestines, urinary tract, eyes, and airways. They are frequent causes of pink eye, diarrhoea, coughs, sore throats, and fever.
  • Statement 2 is correct: Common cold is caused by adenoviruses. HIV is classified as a retrovirus because it contains reverse transcriptase.\
NOTE: Viruses and health issues were in the news since the outbreak of Covid pandemic in 2020.

 

Question 5

With reference to recent developments regarding ‘Recombinant Vector Vaccine’, consider the following statements:

  1. Genetic engineering is applied in the development of these vaccines.

  2. Bacteria and viruses are used as vectors.

Which of the statements given above is/are correct?

(a) 1 only

(b) 2 only

(c) Both 1 and 2

(d) Neither 1 nor 2

Ans: c

Sub-Theme: Vaccines and drugs

  • Recombinant vector vaccines are live replicating viruses that have been modified to carry additional genes taken from a pathogen; these additional genes produce the proteins we want to elicit immunity against.
    • The recombinant virus vector vaccine is one of the important emerging
  • Statement 1 is correct: The techniques of genetic engineering include creation of recombinant DNA, use of gene cloning and gene transfer.
  • Statement 2 is correct: Recombinant vector vaccines expose the body to proteins made by a virus or bacteria, and are often made by using weakened or inactive versions of that virus or bacteria.

 

Question 6

Which one of the following is a reason why astronomical distances are measured in light-years?

(a) Distances among stellar bodies do not change.

(b) Gravity of stellar bodies does not change.

(c) Light always travels in a straight  line.

(d) Speed of light is always the same.

Ans: d

Sub-Theme: Space technology

The Correct answer is Option (d):

  • Space distance is measured in terms of light years. Since the speed of light is known to be constant throughout the universe and for high precision, astronomical distances are expressed in terms of light-years. The fastest light can move is in a The symbol for the speed of light in a vacuum, c, is 3×108 m/s.
  • Astronomers calculate the distances of stars and other celestial objects in the time it takes for light to reach us since estimating distances in miles or kilometres is inadequate on the scale of the cosmos.

 

Question 7

In a pressure cooker, the temperature at which the food is cooked depends mainly upon which of the following?

  1. Area of the hole in the lid

  2. Temperature of the flame

  3. Weight of the lid

Select the correct answer using the code given below.

(a) 1 and 2 only

(b) 2 and 3 only

(c) 1 and 3 only

(d) 1, 2 and 3

Ans: d

Sub-Theme: Basic and Everyday Science

  • Statement 1 is correct: The larger hole will lead to an increase in the steam going out leading to a greater reduction in temperature.
  • Statement 2 is correct: Temperature at which the food is cooked in a Pressure cooker is directly proportional to the temperature of the flame.
  • Statement 3 is correct: By increasing the surface pressure on the liquid inside the closed cooker we can raise the cooking temperature. This pressure can be adjusted by changing the weight placed on the top of the cooker lid.

 

Question 8

Consider the following:

  1. Bacteria

  2. Fungi

  3. Virus

Which of the above can be cultured in artificial/synthetic medium?

(a) 1 and 2 only

(b) 2 and 3 only

(c) 1 and 3 only

(d) 1, 2 and 3

Ans: a

Sub-Theme: Basic and Everyday Science

    • The artificial culture media, also referred to as the growth medium, is a setting for the development of many kinds of microorganisms. The agarose and nutrients needed for the microorganism’s growth are present in the culture medium.
    • This medium is first sterilised to prevent contamination, which prevents the growth of undesirable microorganisms and ensures that it only contains the microbes of In artificial mediums, microbes like fungi and bacteria can be cultivated.
    • Statement 1 is correct: Aspergillus fumigatus, Alternaria penicillium notatum, Cladosporium herbarum, and Penicillium notatum were four key fungi for allergology that were cultivated on a pure synthetic media.
    • Statement 2 is correct: Nutrient agar is the media for bacteria that is most frequently Potato dextrose agar and Sabouraud dextrose agar are the two media that are most frequently used for fungus.
    • Statement 3 is incorrect: Viruses are obligate intracellular parasites that can only multiply in living Because viruses have no internal metabolic machinery and must replicate entirely within the host cell,they cannot be cultured in artificial culture conditions. Eukaryotic or prokaryotic host cells that have been infected can be cultivated, expanded, and the growth media can then be removed to obtain the virus

 

Question 9

With reference to street – lighting, how do sodium lamps differ from LED lamps?

  1. Sodium lamps produce light in 360 degrees but it is not so in the case of LED lamps.

  2. As street -lights, sodium lamps have a longer lifespan than LED lamps.

  3. The spectrum of visible light from sodium lamps is almost monochromatic while LED lamps offer significant colour advantages in street lighting.

Select the correct answer using the code given below.

(a) 3 only

(b) 2 only

(c) 1 and 3 only

(d) 1, 2 and 3

Ans: c

Sub-Theme: Basics and Everyday Science

  • Statement 1 is correct: Sodium lamps emit light in all directions i.e, 360 degree. Because at least half of the light must be reflected and redirected to the intended area of illumination, this technique is significantly inefficient. LEDs, however, emit light in all 180 degrees. Since light is frequently needed over a target region, this is usually advantageous (rather than all 360 degrees around the bulb).
  • Statement 2 is incorrect: High Pressure Sodium (HPS) lights or Sodium lamps have a typical lifespan for sodium lights or sodium lamps is in the range of 24,000 hours. At the halfway point of their lifespan, HPS lamps continue to provide 90% of their initial light output, according to American Electric Lighting. Lumen maintenance at the end of its useful life is still very good at 80%. LEDs have a longer lifespan than any other commercially available light source.
  • Statement 3 is correct: Monochromatic light means that the light has a single wavelength, for eg: sodium light. The LED light is not monochromatic because it is made up of different wavelengths, therefore is polychromatic.

 

Question 10

Bisphenol A (BPA), a cause of concern, is a structural/ key component in the manufacture of which of the following kinds of plastics?

(a) Low-density polyethylene

(b) Polycarbonate

(c) Polyethylene terephthalate

(d) Polyvinyl chloride

Ans: b

Sub-Theme: Environmental Science/Everyday Science

The Correct answer is Option (b): About Bisphenol A (BPA):

  • Bisphenol A (BPA) is a chemical produced in large quantities for use primarily in the production of polycarbonate plastics. It is found in various products including shatterproof windows, eyewear, water bottles, and epoxy resins that coat some metal food cans, bottle tops, and water supply pipes.
  • Polycarbonates (PC) are a group of thermoplastic polymers containing carbonate groups in their chemical structures. Polycarbonates used in engineering are strong, tough materials, and some grades are optically transparent. They are easily worked, moulded, and thermoformed. Because of these properties, polycarbonates find many applications

 

Question 11

‘Triclosan’, considered harmful when exposed to high levels for a long time, is most likely present in which of the following?

(a) Food preservatives

(b) Fruit-ripening substances

(c) Reused plastic containers

(d) Toiletries

Ans: d 

Sub-Theme: Everyday Science

The Correct answer is Option (d): About Triclosan:

  • Triclosan is an ingredient added to many consumer products intended to reduce or prevent bacterial contamination. It is added to some antibacterial soaps and body washes, toothpastes, and some cosmetics.
  • Some short-term animal studies have shown that exposure to high doses of triclosan is associated with a decrease in the levels of some thyroid hormones. Other studies have raised the possibility that exposure to triclosan contributes to making bacteria resistant to antibiotics.

 

 

Question 12

Water can dissolve more substances than any other liquid because

(a) It is dipolar in nature

(b) It is a good conductor of heat

(c) It has high value of specific heat

(d) It is an oxide of hydrogen

Ans: a

Sub-Theme: Basic Science

  The Correct answer is Option (a): About Water:

  • Water is called the “universal solvent” because it can dissolve more substances than any other liquid.
  • It is due to water’s chemical composition and physical attributes that it is such an excellent solvent. Water molecules have a polar arrangement of oxygen and hydrogen atoms—one hand (hydrogen) has a positive electrical charge and the other hand(oxygen) has a negative
  • This makes it possible for the water molecule to attract a wide range of other molecular species. Water can develop a strong attraction to another substance, such as salt (NaCl), to the point where it can interfere with the attractive forces holding the sodium and chloride in the salt complex together and dissolve it.
  • In organic chemistry, a dipolar compound or simply dipole is an electrically neutral molecule carrying a positive and a negative charge in at least one canonical

 

2020

 

Question 1

Consider the following statements:

  1. Genetic changes can be introduced in the cells that produce eggs or sperms of a prospective

  2. A person’s genome can be edited before birth at the early embryonic stage.

  3. Human induced pluripotent stem cells can be injected into the embryo of a pig.

Which of the statements given above is/are correct?

(a) 1 only

(b) 2 and 3 only

(c) 2 only

(d) 1, 2 and 3

Ans: d

Sub-Theme: Gene Therapy

  • Gene therapy is the ability to improve genes through the correction of misplaced (mutated) genes or site-specific alterations with therapeutic treatment as the target
  • A gene abnormality that has been identified in a child or embryo can be corrected using a variety of techniques known as gene therapy. In this instance, genes are injected to treat an illness into a person’s cells and
  • Statement 1 is correct: In germline gene therapy, DNA is inserted into the body’s cells that make eggs or sperm, the reproductive cells. This kind of treatment enables the repair of disease-causing gene mutations that are unavoidably inherited from parents to children. Statement 2 is correct: Although germline gene therapy is prohibited, it is conceivable to alter the genome of an embryo(embryonic genetic editing) to prevent the vertical transfer (through inheritance) of specific diseases and medical disorders.
  • Statement 3 is correct: To conduct an experiment and determine which type of human stem cell would survive the best when injected into pig embryos. Intermediate human pluripotent stem cells were the ones that endured the longest and shown the most potential for further development.

 

Question 2

What is the importance of using Pneumococcal Conjugate Vaccines in India?

  1. These vaccines are effective against pneumonia as well as meningitis and sepsis.

  2. Dependence on antibiotics that are not effective against drug resistant bacteria can be reduced.

  3. These vaccines have no side effects and cause no allergic reactions.

Select the correct answer using the code given below:

(a) 1 only

(b) 1 and 2 only

(c) 3 only

(d) 1, 2 and 3

Ans: b

Sub-Theme: Vaccines and drugs

  • By eliciting an immune response to an antigen, a recognisable component of a bacterium or virus, vaccines work to prevent In order to achieve this, a harmful bacterium or virus that is attenuated or dead is typically included in the vaccine. This way, the immune system is trained to detect the antigen in the future.
  • Statement 1 is correct: A pneumococcal vaccine known as a Pneumococcal Conjugate Vaccine (PCV) provides protection against pneumococcal illnesses, which are infections brought on by the pneumococcus, or Streptococcus pneumoniae, bacterium. Ear infections, sepsis, sinus infections, pneumonia, and bloodstream infections are all possible outcomes of pneumococcal infections.
  • A pneumococcal vaccine, often known as a pneumonia shot, protects against septicemia (also known as sepsis), a type of blood poisoning, and meningitis.
  • Statement 2 is correct: Although vaccines are not meant to replace antibiotics, they can help decrease AMR (Antimicrobial Resistance or drug resistance) by decreasing the spread of (resistant) bacterial illnesses as well as by lowering the overuse and misuse of antibiotics. Statement 3 is incorrect: The side effects of the pneumococcal conjugate vaccine might range from fever, appetite loss, and fussiness to headache.
Note: Read statement 3 carefully. It talks about “no side effects and no allergic reactions”. It sounds like a very absolute and extreme statement. Any drug has its own side effects with variable magnitude. Allergic reactions vary person to person. Drugs having zero side effects and allergic reactions are unconvincing. So elimination of statement 3 will help us to reach the correct options.

 

Question 3

With the present state of development, Artificial Intelligence can effectively do which of the following?

  1. Bring down electricity consumption in industrial units

  2. Create meaningful short stories and songs

  3. Disease diagnosis

  4. Text -to -Speech Conversion

  5. Wireless transmission of electrical energy

Select the correct answer using the code given below:

(a) 1, 2, 3 and 5 only

(b) 1, 3 and 4 only

(c) 2, 4 and 5 only

(d) 1, 2, 3, 4 and 5

Ans: b

Sub-Theme: Artificial Intelligence

  • Artificial intelligence (AI) is the ability of machines to carry out cognitive functions like reasoning, perception, learning, and problem-solving.
  • Intelligent systems may now be used to handle a range of activities, facilitate communication, and boost productivity thanks to significant advancements achieved in data gathering, processing, and computation power.
  • In today’s culture, artificial intelligence is used in a variety of ways. In a variety of sectors, including healthcare, entertainment, finance, and education, among others.
  • AI has been used to diagnose diseases, write songs. Text-to-speech conversion has made use of AI, such as Cerewave AI.
  • For instance, Google is combining the Internet of Things (IoT) and artificial intelligence (AI) from its acquisition of DeepMind to cut the amount of energy used in its data centres by up to 30%.

 

Question 4

With reference to Visible Light Communication (VLC) technology, which of the following statements are correct? 

  1. VLC uses electromagnetic spectrum wavelengths 375 to 780 nm

  2. VLC is known as long – range optical wireless

  3. VLC can transmit large amounts of data faster than Bluetooth

  4. VLC has no electromagnetic interference

Select the correct answer using the code given below:

(a) 1, 2 and 3 only

(b) 1, 2 and 4 only

(c) 1, 3 and 4 only

(d) 2, 3 and 4 only

Ans: c

Sub-Theme: Communication technology

Visible Light Communication (VLC)

  • It uses visible light between 400 and 800 THz (375–780 nm) for data Optical wireless communications methods include VLC as a subset.
  • The light can communicate information directly or indirectly through surface reflection. Even in low light, it can still function. Light, however, cannot pass through solid objects like walls. Therefore, VLC cannot be utilised for long distance wireless transmission.
  • VLC can send data at speeds between mbps and gbps, but Bluetooth can only carry data at a rate of 300kbps (kilobytes per second) (megabytes per second to gigabytes per second).
    • With the introduction of LED, general lighting is now being used by VLC as an alternative or addition to Wi-Fi to connect with people within a location. LED illumination can be adjusted at very high frequencies with a cycle as short as nanoseconds, but other light sources have practical constraints.
  • Due to the fact that the visible light spectrum is 10,000 times bigger than the radio spectrum, this idea, known as Li-Fi, may provide a solution to RF bandwidth restrictions. VLC possesses the ability to withstand electromagnetic As a result, there is no electromagnetic interference.
  • Because light-producing equipment (such as indoor/outdoor lamps, TVs, traffic signs, commercial displays, and car headlights/ taillights) are utilised everywhere, VLC can be employed as a communications channel for ubiquitous computing.
  • Statement 1 is correct: Visible light communication (VLC) is a data communications variant which uses visible light between 400 and 800 THz (375–780 nm). VLC is a subset of optical wireless communications technologies. Statement 2 is incorrect: The light can transmit information either directly or reflected from a surface. It can do so while dimmed. However, light cannot penetrate obstacles such as walls. Thus for wireless communication purposes VLC cannot be used for long distances.
  • Statement 3 is correct: Bluetooth transmits data at the rate of 300kbps (kilobytes per second), while VLC can transmit data in the range between mbps -gbps (megabytes per second to gigabytes per second).
  • Statement 4 is correct: VLC has the characteristic to resist electromagnetic interference. So it does not cause Electromagnetic interference.

 

Question 5

With reference to “Blockchain Technology” consider the following statements:

  1. It is a public ledger that everyone we inspect, but which no single user controls

  2. The structure and design of blockchain is such that all the data in it is about crypto currency only.

  3. Applications that depend on basic features of blockchain can be developed without anybody’s

Which of the statements given above is/are correct?

(a) 1 only

(b) 1 and 2 only

(c) 2 only

(d) 1 and 3 only

Ans: d

Sub-Theme: Blockchain Technology

Blockchain Technology

  • A blockchain is a digital ledger that is decentralised, distributed, and frequently made public. It is used to record transactions across many computers so that any involved block cannot be changed retroactively without changing all succeeding blocks.

This makes it possible for participants to independently and reasonably audit transactions. A distributed timestamping server and a peer-to-peer network are used to manage a blockchain database

    • Blockchain technology can be applied to supply chains, healthcare, smart contracts, banking, real estate transactions, and even election processes.
    • The main benefit of an open, permissionless, or public blockchain network is that access management and security against malicious actors are not This means that by leveraging the blockchain as a transport layer, apps can be introduced to the network without needing the consent or confidence of others.
  • Statement 1 is correct: A blockchain is a decentralised, distributed, and oftentimes public, digital ledger consisting of records called blocks that is used to record transactions across many computers so that any involved block cannot be altered retroactively, without the alteration of all subsequent blocks.
  • Statement 2 is incorrect: Blockchain technology can be used in property exchanges, bank transactions, healthcare, smart contracts, supply chain, and even in voting for a candidate
  • Statement 3 is correct: The great advantage to an open, permissionless, or public, blockchain network is that guarding against bad actors is not required and no access control is needed.
Note: With general understanding of Blockchain technology through Newspapers/ Monthly magazines, we know that this technology has uses and applications across various sectors such as in banking sector and health sector among others. Statement 2 talks about the application of Blockchain technology for crypto currency only, which is an extreme and absolute statement. Eliminating statement 2 by applying the above understanding helps us to reach at 50:50 probability.

 

Question 6

When reference to carbon nanotubes, consider the following statements:

  1. They can be used as carriers of drugs and antigens in the human body.

  2. They can be made into artificial blood capillaries for an injured part of the human body.

  3. They can be used in biochemical sensors.

  4. Carbon nanotubes are biodegradable.

Which of the statements given above are correct?

(a) 1 and 2 only

(b) 2, 3 and 4 only

(c) 1, 3 and 4 only

(d) 1, 2, 3 and 4

Ans: c

Sub-Theme: Nanotechnology

Carbon nanotubes (CNTs)

  • CNTs are cylindrical molecules which consist of rolled-up sheets of single-layer carbon atoms (graphene).
  • They come in two varieties: single-walled (SWCNT), with a diameter of less than

1 nanometer (nm), and multi-walled (MWCNT), with diameters up to more than 100 nm and made up of multiple concentrically interconnected nanotubes. They can be as long as a millimetre or even several micrometres.

  • Due to their large surface area, carbon nanotubes may be delivered to a particular cell with a very high degree of efficiency because they enter cells in
  • Since carbon nanotubes are promising drug delivery platforms that may be functionalized
  • with a range of biomolecules, such as antibodies, proteins, or DNA, they can be formed into prosthetic blood capillaries for a damaged region of the human body.
    • NASA has shown off Biochemical Sensors Made of Carbon Nanotube Arrays. Using incorporated vertically aligned carbon nanotubes as nanoelectrode arrays in diagnostic devices, NASA has successfully shown a miniaturised electronics technology with extraordinarily high sensitivity and simplified sample preparation for in-vitro detection of specific biomarker signatures.
    • Carbon nanotubes can be broken down by a variety of microorganisms, including bacteria and fungi (CNTs).
  • Statement 1 is correct: carbon nanotubes have a high surface area, they go into the cell by the millions, and can have a very high efficiency of delivery to a specific cell
  • Statement 2 is incorrect: Carbon nanotubes cannot be made into artificial blood capillaries for an injured part of the human body as they are promising drug delivery platforms that can be functionalized with a variety of biomolecules, such as antibodies, proteins, or DNA.
  • Statement 3 is correct: NASA has successfully demonstrated a  miniaturized  electronics technology with extremely high sensitivity and simplified sample preparation for in-vitro detecting specific biomarker signatures, which is based on incorporating embedded vertically aligned carbon nanotubes as nanoelectrode arrays in diagnostics devices.
  • Statement 4 is correct: Multiple types of microbes, including bacteria and fungi, have the ability to degrade carbon nanotubes (CNTs)

 

Question 7

The experiment will employ a trio of spacecraft flying in formation in the shape of an equilateral triangle that has sides one million kilometers long, with lasers shining between the craft.” The experiment in question refers to

(a) Voyager – 2

(b) New Horizons

(c) LISA Pathfinder

(d) Evolved LISA

Ans: d

Sub-Theme: Space technology/Space mission

The Correct answer is Option (d):

  • The evolved Laser Interferometer Space Antenna (eLISA) mission aims to conduct the first-ever space-based study of the gravitational universe. It includes researchers from eight different European nations, including Denmark, France, Germany, Italy, The Netherlands, Spain, Switzerland, and the United Kingdom, as well as some US-based ones.
  • The European Space Agency accepted the “The Gravitational Universe” subject (with eLISA as the anticipated implementation) and chose it as a science theme for the third large-class mission to be launched in 2034 as part of the agency’s Cosmic Vision research programme.
  • Two “Daughter” spacecraft and a “Mother” spacecraft make up the eLISA mission. These will form a triangle orbit around the Sun. The two Daughter spacecraft will be joined to the Mother satellite, and the three satellites will form a precision interferometer by 1 million km long laser

 

Question 8

Consider the following activities:

  1. Spraying pesticides on a crop field

  2. Inspecting the craters of active volcanoes

  3. Collecting breath samples from spouting whales for DNA analysis.

At the present level of technology, which of the above activities can be successfully carried out by using drones?

(a) 1 and 2 only

(b) 2 and 3 only

(c) 1 and 3 only

(d) 1, 2 and 3

Ans: d

Sub-Theme: Applications of Drone Technology

The Correct answer is Option (d): About Drone technology:

  • A drone, technologically, is an unmanned aircraft. Drones are formally known as unmanned aerial vehicles (UAVs).
  • A drone is essentially a flying robot that can be remotely piloted or can fly on its own using software-controlled flight plans in embedded systems in conjunction with onboard sensors and GPS.
  • In Telangana, insecticides were sprayed using drones, which made In India, it is regarded as unlawful. Aerial spraying is prohibited by the Insecticide Act of 1968. Aerial pesticide application is prohibited by its regulations unless authorised by the Central Insecticides Board (CIB). However, many states in the US allow for its legal practice. (Statement 1 is correct)
  • Drones have also been used to explore the craters of active volcanoes, which are inaccessible to people, in order to learn about their recent activities.(Statement 2 is correct)
  • Recently, Australian scientists used drones (that flew 200 metres above the blowholes of whales) that collected the mucus of whales from their water sprays for examining their health. Those whale spray collected by a drone contains DNA, proteins, lipids and types of bacteria. (Statement 3 is correct)

 

Question 9

Which of the following statements are correct regarding the general difference between plant and animal cells?

  1. Plant cells have cellulose cell walls whilst animal cells do not.

  2. Plant cells do not have plasma membranes unlike animal cells which do.

  3. Mature plant cell has one large vacuole whilst an animal cell has many small vacuoles.

Select the correct answer using the code given below:

(a) 1 and 2 only

(b) 2 and 3 only

(c) 1 and 3 only

(d) 1, 2 and 3

Ans: c

Sub-Theme: Basic and Everyday Science

  • Statement 1 is correct: Both plants and animals contain eukaryotic cells, their cell structures  are  quite  similar.  The  plasma membrane, nucleus, cytoplasm, ribosomes, and mitochondria in general make up each eukaryotic cell.The plant cell differs significantly from other cells in that it has a hard cell wall surrounding it. While the cells of other species are surrounded by a cell wall, cellulose and proteins are present in plant cells.
  • Statement 2 is incorrect: Both plants and animals contain eukaryotic cells due to this their cell structures are quite similar. The plasma membrane, nucleus, cytoplasm, ribosomes, and mitochondria in general make up each eukaryotic cell.
  • Statement 3 is correct: Large central vacuoles can be found in plant cells. Similar to several other eukaryotes, plant cells have bigger structures called vacuoles. Animal cells, which are much smaller than plant cells, may contain numerous tiny vacuoles.

 

Question 10

In the context of recent advances in human reproductive technology. “Pronuclear Transfer” is used for

(a) Fertilisation of egg in vitro by the donor sperm

(b) Genetic modification of sperm producing cells

(c) Development of stem cells into functional embryos

(d) Prevention of mitochondrial diseases in offspring

Ans: d

Sub-Theme: Human Reproductive Technology

The Correct answer is Option (d): About Pronuclear Transfer:

  • Mitochondrial transfer methods employ proton In pronuclear transfer, a zygote is created by first fertilising the mother’s egg with the father’s sperm.
  • The pronuclei of the egg and sperm are then removed from the zygote and inserted into a donor egg which has been fertilised and had its own nucleus removed (a pronucleus is the nucleus of the egg or sperm at the stage of fertilisation prior to nuclear fusion). Thezygote derived from the donor egg is then inserted into the mother’s uterus.
    • The pronuclei of zygotes produced from donated oocytes are extracted from a karyoplast using micromanipulation equipment and thrown away.
    • Two pronuclei from the afflicted zygotes (also in the form of a karyoplast) are transferred into the enucleated healthy zygotes during a therapeutic pronuclear transfer. The resultant zygotes have mtDNA from a donor and nuclear DNA from each of the intended parents.

 

Question 11

With reference to solar water pumps, consider the following statements:

  1. Solar power can be used for running surface pumps and not for submersible pumps.

  2. Solar power can be used for running centrifugal pumps and not the ones with piston.

Which of the statements given above is/are correct?

(a) 1 only

(b) 2 only

(c) Both 1 and 2

(d) Neither 1 nor 2

Ans: d

Sub-Theme: Solar water pumps

  • Statement 1 is incorrect: An electrical pump system that uses one or more photovoltaic (PV) panels to generate power is essentially what a solar water pump system is. A solar panel array often drives an electric motor, which in turn powers a bore or surface pump in a solar-powered pumping system. Depending on its operational mechanism, solar-powered pumps come in a variety of distinct forms. Submersible pumps, surface pumps, direct current (DC) pumps, and alternative current (AC) pumps are the four main categories of solar water pumps.
  • Statement 2 is incorrect: Centrifugal and piston pumps can both be driven by solar power. Centrifugal pumps, multistage pumps, borehole pumps, and helical pumps are the most often utilised pump mechanics in solar power pumps.

 

Question 12

With reference to the current trends in the cultivation of sugarcane in India, consider the following statements:

  1. A substantial saving in seed material is made when ‘bud chip settings’ are raised in a nursery and transplanted in the main field.

  2. When direct planting of seeds is done, the germination percentage is better with single- budded setts as compared to setts with many

  3. If bad weather conditions prevail when seeds are directly planted, single – budded seeds have better survival as compared to large setts.

  4. Sugarcane can be cultivated using settlings prepared from tissue culture.

Which of the statements given above is/are correct?

(a) 1 and 2 only

(b) 3 only

(c) 1 and 4 only

(d) 2, 3 and 4 only

Ans: c         

Sub-Theme: Science related to agriculture

The Correct answer is Option (c):

About the cultivation of sugarcane in India:

  • Tissue culture (TC) is the cultivation of plant cells, tissues, or organs on specially formulated nutrient Under the right conditions, an entire plant can be regenerated from a single cell.
  • Commercial planting of sugarcane uses setts (cuttings that can form roots) at a rate of 6 to 8 tonnes per hectare, or around 10% of total production.
  • Due to its bulky attire and physical restrictions, the tissue culture method is becoming uneconomical. Planting excised axillary canestalk buds, sometimes referred to as budchips, would help reduce the mass and enhance the quality of the seed cane.
  • After four to five weeks, healthy settlings were transplanted on a well-prepared field with a row spacing of 90 cm and a gap of 30 cm between each settling (small setts). (Statement 1 is correct)
    • Due to moisture loss from the other cut end, the germination percentage of a single bud sett is extremely Numerous studies and trials demonstrate that the germination rate of 3 bud sets is higher than the setts having more or less than three buds. (Statement 2 is incorrect)
    • Additionally, even if the entire can stem is planted without being trimmed, only the top end will sprout, resulting in a poor germination percentage.
    • When seeds are sown directly in the ground during severe weather circumstances, huge setts of seeds do better than single budded seeds in terms of survival. (Statement 3 is incorrect)
    • The most crucial and labor-intensive step in sugarcane agriculture is planting. Supplying disease-free seed cane of current commercial types quickly an innovative new technique called tissue culture employs meristems to clone the mother Tissue culture plants’ cane and sugar yields are comparable to those of plants grown using traditional methods. (Statement 4 is correct)

 

2019

 

Question 1

With reference to the recent developments in science, which one of the following statements is not correct?

(a) Functional chromosomes can be created by joining segments of DNA taken from cells of different species.

(b) Pieces of artificial functional DNA can be created in laboratories.

(c) A piece of DNA taken out from an animal cell can be made to replicate outside a living cell in a

(d) Cells taken out from plants and animals can be made to undergo cell division in laboratory petri dishes

Ans: a

Sub-Theme: Genetics/DNA

  • Statement 1 is incorrect: The combining of DNA fragments acquired from cells of different species cannot result in the creation of functional chromosomes.
  • Statement 2 is correct: In a lab, artificial functioning DNA fragments can be produced. Japanese researchers announced the creation of the first DNA molecule in the world that is almost entirely formed of synthetic components in 2007.
  • Statement 3 is correct: In a lab, it is possible to get a portion of animal DNA to reproduce on its own outside of a living cell. It is done through the Polymerase Chain Reaction (PCR) technology.
  • Statement 4 is correct: In a lab setting, Petri dishes can be used to induce cell division in isolated plant and animal cells. Plant cells are totipotent and a single cell has the capacity to divide through the process of mitosis in sterile conditions in a culture Petri dish to develop into a mature plant.

 

Question 2

‘RNA interference (RNAi)’ technology has gained popularity in the last few Why?

  1. It is used in developing gene silencing therapies.

  2. It can be used in developing therapies for the treatment of cancer.

  3. It can be used to develop hormone replacement therapies.

  4. It can be used to produce crop plants that are resistant to viral pathogens.

Select the correct answer using the code given below.

(a) 1, 2 and 4

(b) 2 and 3

(c) 1 and 3

(d) 1 and 4 only

Ans: a

Sub-Theme: Genetics/Biotechnology

RNA interference (RNAi) or Post- Transcriptional Gene Silencing (PTGS) is a conserved biological response to double- stranded RNA that mediates resistance to both endogenous parasitic and exogenous pathogenic nucleic acids, and regulates the expression of protein-coding genes.

  • Statement 1 is correct: Its natural mechanism for sequence-specific gene silencing promises to revolutionize experimental biology and may have important practical applications in functional  genomics, therapeutic intervention, agriculture and other areas.
  • Statement 2 is correct: Numerous studies have demonstrated that RNAi can provide a more specific approach to inhibit tumor growth by targeting cancer-related genes (i.e., oncogene).
  • Statement 3 is incorrect: RNA interference (RNAi) cannot be used to develop hormone replacement therapies.
  • Statement 4 is correct: The reverse experiment “virus-induced gene silencing” in which short sequences of plant genes were introduced into viruses, showed that the targeted gene was suppressed in an infected plant. It will produce crop plants that are resistant to viral pathogens.

 

Question 3

Which of the following are the reasons for the occurrence of multi-drug resistance in microbial pathogens in India?

  1. Genetic predisposition of some people

  2. Taking incorrect doses of antibiotics to cure diseases

  3. Using antibiotics in livestock farming

  4. Multiple chronic diseases in some people

Select the correct answer using the code given below.

(a) 1 and 2

(b) 2 and 3 only

(c) 1, 3 and 4

(d) 2, 3 and 4

Ans: b

Sub-Theme: Multi-drug resistance

  • Rising incomes, decreased prescription costs, and uncontrolled sales have resulted in an increase in antibiotic use and higher rates of resistance, evaluations of the burden of resistance have received little attention in low- and middle-income countries (LMICs)
  • Statement 1 is incorrect: genetic predisposition is a genetic trait that, in response to environmental factors, may influence the potential phenotypic development of an individual organism within a species or community.
    • Antibiotic resistance is unrelated to the genetic tendency of a small minority of persons in a society.
  • Statement 2 is correct: The rise of bacterial strains that are resistant to several antibiotics could be caused by the improper usage of antibiotic FDCs.
  • Statement 3 is correct: Using the antibiotics in livestock farming increases, the antibiotic multidrug resistance transfers in microbial pathogens
  • Statement  4  is  incorrect:  Additionally,MDR exacerbates the risk of resistant microorganisms spreading, which reduces treatment effectiveness and, ultimately, prolongs the duration of an infection in patients. But, multiple chronic diseases in people is not the reason of MDR.

 

Question 4

What is Cas9 protein that is often mentioned in the news?

(a) A molecular scissors used in targeted gene editing

(b) A biosensor used in the accurate detection of pathogens in patients

(c) A gene that makes plants pest-resistant

(d) A herbicidal substance synthesized in genetically modified crops

Ans: a

Sub-Theme: Gene editing

Option (a) is correct: The Cas9 enzyme functions as a molecular scissor that can cut DNA, and the single-guide RNA (sgRNA) contains a sequence that can bind to DNA. Together, these two components make up the gene editing tool

About CRISPR Cas9:

  • Clustered Regularly Interspaced Short Palindromic Repeats” or CRISPR Cas9 is the abbreviation.
  • With the use of CRISPR genome editing technology, researchers may quickly and precisely alter any genome’s DNA.
  • As a sort of adaptive immunity, bacteria use the CRISPR-Cas9 system to identify and eliminate viral DNA. Researchers may add, delete, or modify specific DNA sequences from the genomes of higher species using the CRISPR system’s components.
  • The Cas9 enzyme functions as a molecular scissor that can cut DNA, and the single- guide RNA (sgRNA) contains a sequence that can bind to DNA. Together, these two components make up the gene editing

 

Question 5

Which one of the following statements is not correct?

(a) Hepatitis B virus is transmitted much like HIV.

(b) Hepatitis B, unlike Hepatitis C, does not have a vaccine.

(c) Globally, the number of people infected with Hepatitis B and C viruses are several times more than those infected with HIV.

(d) Some of those infected with Hepatitis B and C viruses do not show the symptoms for many years.

Ans: b

Sub-Theme: Vaccines and drugs

Option (b) is correct: There is currently no cure for Hepatitis C. The mainstay of hepatitis B prevention is the hepatitis B vaccine.

  • A viral infection known as hepatitis B affects the liver and can result in both acute and chronic illness. Contact with the blood or other bodily fluids of an infected individual can spread the virus.
  • HIV, Hepatitis B, and Hepatitis C can all be spread through unprotected sexual contact, contaminated syringes, infected blood, and
  • Hepatitis B and C afflict a far greater number of people than HIV infection does.
  • There is currently no cure for Hepatitis C. The mainstay of hepatitis B prevention is the hepatitis B vaccine.
  • The most prevalent serious liver infection in the world and a hazard to public health globally is hepatitis B. It can spread up to 100 times faster than the HIV/AIDS virus.
  • Hepatitis B and C infections can cause chronic liver disease, liver cirrhosis, and hepatocellular cancer in people who are symptom-free for years or who only experience minor symptoms.

 

Question 6

With reference to communications technologies, what is/are the difference/differences between LTE (Long- Term Evolution) and VoLTE (Voice over Long-Term Evolution)?

  1. LTE is commonly marketed as 3G and VoLTE is commonly marketed as advanced 3G.

  2. LTE is data-only technology and VoLTE is voice- only technology.

Select the correct answer using the code given below.

(a) 1 only

(b) 2 only

(c) Both 1 and 2

(d) Neither 1 nor 2

Ans: d

Sub-Theme: Communication Technology

  • The LTE protocol used by mobile phone networks has undergone a technological update called Voice over LTE (VoLTE). Voice calls are routed to the telecom companies’ older 2G or 3G networks under LTE, which solely supports data transmission.
  • Because of this, using LTE prevents you from using your 4G data services while on a call. This causes issues like sluggish internet speeds and muddled voice quality.
  • The most popular version of Long Term Evolution (LTE) is 4G LTE.
    • VoLTE enables the packaging and transmission of voice calls over LTE networks. This indicates that 4G data is usable even while on a call.
    • VoLTE is an Internet Protocol Multimedia Subsystem (IMS) specification that allows for smooth network operation of a number of services, such as voice and video, without the need to switch between different
  • This would provide access to 4G data even when on a call.
  • Statement 1 is incorrect: Long term Evolution (LTE) is commonly marketed as 4G LTE.
  • Statement 2 is incorrect: VoLTE is an Internet Protocol Multimedia Subsystem (IMS) specification which enables a variety of services to operate seamlessly on the network rather than having to switch to different applications for voice or video.
NOTE: UPSC simply loves to ask questions on new age technologies which are frequent in news. You need not know every single intricacies but UPSC expects the aspirants to know ‘something’ of ‘everything’ under the sun. Therefore, it is advisable to read the daily newspaper and understand the basic concepts of those technologies, through additional research, for eg: 5G, AI, Blockchain, Additive Manufacturing, 3D Printing, Deep fakes, Brain interface, Augmented Reality, Metaverse, etc.

 

Question 7

In the context of digital technologies for entertainment, consider the following statements:

  1. In Augmented Reality (AR), a simulated environment is created and the physical world is completely shut out.

  2. In Virtual Reality (VR), images generated from a computer are projected onto real -life objects or

  3. AR allows individuals to be present in the world and improves the experience using the camera of smart – phones or PCs.

  4. VR closes the world, and transposes an individual, providing a complete immersion experience.

Which of the statements given above is/are correct?

(a) 1 and 2 only

(b) 3 and 4

(c) 1, 2 and 3

(d) 4 only

Ans: b

Sub-Theme: Digital Technologies

The main difference between the two is that VR(Virtual Reality) builds the world in which we immerse ourselves through a specific headset. It is fully immersive and everything we see is part of an environment artificially constructed through images, sounds, etc. On the other hand, in augmented reality (AR), our own world becomes the framework within which objects, images or similar are placed.

  • Statement 1 is incorrect: Users view and interact with the physical environment while digital content is added to in augmented reality (AR). A good example of augmented reality is Pokemon Go.
  • Statement 2 is incorrect: In Virtual Reality (VR), images generated from a computer are not projected onto real -life objects or surroundings rather, with help of virtual reality headset, helmet, or other equipment a virtual reality (VR) environment gives the user the impression that they are completely engrossed in their surroundings.
  • Statement 3 is correct: Users may easily test out AR with the camera of a smartphone by downloading an app. However, there is another way to experience augmented reality. With specialised AR headsets, like Google Glass, digital content is displayed on a small screen in front of the user’s eyes.
  • Statement 4 is correct: With images and things that seem real, a virtual reality (VR) environment gives the user the impression that they are completely engrossed in their surroundings. A virtual reality headset, helmet, or other equipment is used to view this environment.

 

Question 8

Consider the following statements:

A digital signature is __________

  1. an electronic record that identifies the certifying authority issuing it

  2. used to serve as a proof of identity of an individual to access information or serve on Internet

  3. an electronic method of signing an electronic document and ensuring that the original content is unchanged

Which of the statements given above is/are correct?

(a) 1 only

(b) 2 and 3 only

(c) 3 only

(d) 1, 2 and 3

Ans: c

Sub-Theme: Digital technology

  • A digital signature is a type of electronic signature that can be used to verify the identity of the person sending a message or signing a document and to guarantee that the message’s or document’s original content hasn’t changed.
  • Digital signatures are portable and impossible for anyone to forge. The sender cannot just deny the message later if it can be verified that the Original Signed message actually arrived.
  • Certificates serve as evidence of a person’s identity for specific purposes; for instance, a driver’s licence identifies someone who is legally permitted to drive in a specific
  • Similarly, you can use a digital certificate to sign specific papers electronically or to authenticate your identity while accessing information or services online.
  • However, compared to digital signature certificates, digital signatures are distinct (DSC). DSC is the electronic version of paper certificates like a passport or driver’s licence. A digital signature certificate is a computer-based record that identifies the certifying authority providing it, while a digital signature is an electronic technique of conveying an electronic document.
    • Statement 1 is incorrect: The digital signature is an electronic method of sending an electronic document whereas a digital signature certificate is a Computer Based record that identifies the certifying authority issuing it.
    • Statement 2 is incorrect: A digital certificate is not used to prove your identity, to access information or services on the Internet or to sign certain documents digitally.
    • Statement 3 is correct: A digital signature is an electronic form of signature that can be used to authenticate the Identity of the sender of a message or a signer of a document and also ensure that original content of the message on document that has been sent is unchanged.

 

Question 9

In the context of wearable technology, which of the following tasks is/are accomplished by wearable devices?

  1. Location identification of a person

  2. Sleep monitoring of a person

  3. Assisting the hearing impaired person

Select the correct answer using the code given below.

(a) 1 only

(b) 2 and 3 only

(c) 3 only

(d) 1, 2 and 3

Ans: d

Sub-Theme: Digital Technology

  • Electronic technologies or devices that may be comfortably worn on the body are known as wearables. Real-time information tracking is done with these wearable
  • They have motion sensors that capture an image of your daily activity and sync it with laptops or mobile devices. Wearable electronics are the next major technological advance after the introduction of smartphones.
  • Most wearable devices, including activity trackers, smartwatches, and even medical wearables that monitor patients’ physical activity and health, include location and position-tracking sensors, such as GPS, altimeters, magnetometers, compasses, and accelerometers.
  • Fitness trackers, often referred to as activity trackers, are typically worn on the wrist, chest, or ears and are intended to track outdoor sporting activities as well as assess fitness-related parameters including running speed and distance, breath, heart rate, and sleeping patterns.
  • Hearables, or wearable technology worn on the ear, have recently experienced a fresh wave of popularity. The type of hearables known as Hearing-Aids uses a microphone, speaker, and This enhances noises to improve hearing for those who have hearing loss. as in the Phonak Audeo V
    • Statement 1 and 2 are correct: Wearables are electronic technology or devices incorporated into items that can be comfortably worn on a body. These wearable devices are used for tracking information such as location, sleep data, fitness track, etc. on a real-time basis.
    • Statement 3 is correct: The type of hearables known as Hearing-Aids enhances noises to improve hearing for those who have hearing loss.

 

Question 10

For the measurement/ estimation of which of the following are satellite images/remote sensing data used?

  1. Chlorophyll content in the vegetation of a specific location.

  2. Greenhouse gas emissions from rice paddies of a specific location.

  3. Land surface temperatures of a specific

Select the correct answer using the code given below.

(a) 1 only

(b) 2 and 3 only

(c) 3 only

(d) 1, 2 and 3

Ans: d

Sub-Theme: Remote sensing technology/Space technology

For assessing chlorophyll content from remote sensing data utilising a multispectral imaging system, the Normalized Area Vegetation Index (NAVI) has been developed. Using satellite photography and the Normalized Difference Vegetation Index, the world’s food supply is being tracked (NDVI).

  • In agriculture, near-infrared radiation is utilised to identify healthy vegetation. Green light is reflected by healthy vegetation, while red and blue light are absorbed. Our eyes perceive green light because plants produce chlorophyll during photosynthesis.
  • Remote sensing technology provides more accurate paddy field data. Resolution Imaging Spectroradiometer is used to estimate GHGs emissions like CH4 emissions from rice paddies.
  • The most widely used instruments are radiometers and photometers, which can collect radiation that is emitted and reflected at a variety of frequencies. Additionally, they can be used to identify the emission spectra of different substances, providing information on atmospheric chemical
  • One of the most important factors in land- surface physical processes at the regional and global levels is land surface temperature (LST), which has been extensively used in hydrology, meteorology, and the surface energy balance. A distinctive method for determining the LST at both regional and global scales is remote sensing.
  • Statement 1 is correct: Chlorophyll will reflect more light in the green and near infrared spectrum compared to other wavelengths. This is why near infrared radiation in combination with NDVI is one of the primary remote sensing applications in agriculture and the environment. Statement 2 is correct: Remote sensing technology is capable of providing more accurate paddy field data. Resolution Imaging Spectroradiometer is used to estimate Greenhouse gas emissions like CH4 emissions from rice paddies.
  • Statement 3 is correct: Remote sensing is a unique way of obtaining the Land Surface Temperature (LST) at regional and global scales.

 

Question 11

Recently, scientists observed the merger of giant ‘blackholes’ billions of light years away from the Earth. What is the significance of this observation?

(a) ‘Higgs boson particles’ were detected.

(b) ‘Gravitational waves’ were detected.

(c) Possibility of intergalactic space travel through ‘wormhole’ was confirmed.

(d) It enabled the scientists to understand ‘singularity’.

Ans: b

Sub-Theme: Space science/Gravitational waves

The Correct answer is Option (b):

  • The LIGO detectors detected the first gravitational waves produced by two giant merging blackholes in 2016.
  • In 2017, the Nobel Prize for physics was awarded to the LIGO-VIRGO collaboration for their detection of gravitational waves arising from the merging of two black
  • Gravitational waves are ripples in spacetime’s fabric brought on by cosmic catastrophes like neutron star or black hole collisions. These disturbances are incredibly powerful, yet their signals are feeble and difficult to detect when they travel through distant regions of space and time.
  • In other words, it took that long for the signals to reach Earth. As a result, the observatory provides a chance to go back in time and solve puzzles about the origins of the universe.
  • The LIGO-VIRGO collaboration has found these signals four times since then. Black holes, neutron stars, and other similar objects may be mapped using gravitational wave astronomy, which is currently a science of the near future. Astronomy provides a technique of mapping the visible objects in the universe.
  • A wormhole is a hypothetical structure that connects dispersed places in spacetime. It is based on a unique solution to the Einstein field equations. It is also known as an Einstein-Rosen bridge or wormhole. Although wormholes are consistent with the general theory of relativity, it is yet unknown whether they truly exist.
  • The Higgs field, one of the fields in particle physics theory, undergoes quantum excitation to form the Higgs boson, an elementary particle in the Standard Model of particle physics. The Higgs particle is a large scalar boson with no electric charge, colour charge, or spin in the Standard Model.
  • The technological singularity, often known as the singularity, is a hypothetical time period in which unpredictably drastic changes to human civilisation result through uncontrollable and irreversible technology advancement. John von Neumann was the first to use the term “singularity” in a technological context.

 

Question 12

Consider the following statements:

  1. According to the Indian Patents Act, a biological process to create a seed can be patented in

  2. In India, there is no Intellectual Property Appellate

  3. Plant varieties are not eligible to be patented in

Which of the statements given above is/are correct?

(a) 1 and 3 only

(b) 2 and 3 only

(c) 3 only

(d) 1, 2 and 3

Ans: c

Sub-Theme: Patent

  • Statement 1 is incorrect: As per the Patents Act, 1970 – “plants and animals in whole or any part thereof other than micro organisms but including seeds, varieties and species and essentially biological processes for production or propagation of plants and animals” are inventions not patentable.
  • Statement 2 is incorrect: In India, the Intellectual  Property  Appellate  Board, a quasi -judicial body, was constituted in September 1958. The Board is given the task of adjudication of disputes related to copyright registration, assignment of copyright, grant of Licences in respect of works withheld from public, unpublished Indian works, production and publication of translations and works for certain specific purposes. It also hears cases in other miscellaneous cases instituted before it under the Copyright Act, 1957.
  • Statement 3 is correct: There are no laws in India that allow for patenting of plants. The Delhi High Court invalidated Monsanto’s patent on Bt cotton seeds in April 2018. In its ruling, the two-judge panel presided by Justice S. Ravindra Bhat stated that some things, including seeds, animals, and plants, cannot be patented in India. However, the Supreme Court recently permitted agricultural juggernaut Monsanto to claim patent on its genetically modified (GM) cotton seeds by overturning a Delhi High Court decision on January 8, 2019.

 

2018

 

Question 1

Consider the following

  1. Birds

  2. Dust blowing

  3. Rain

  4. Wind blowing

Wind blowing Which of the above spread plant diseases?

(a) 1 and 3 only

(b) 3 and 4 only

(c) 1, 2 and 4 only

(d) 1, 2, 3 and 4

Ans: d

Sub-Theme: Plant diseases

Option (d) is correct: Plant disease is an alteration to a plant’s normal state that impairs or changes its essential functioning.

  • All plant species, both wild and domesticated, are susceptible to illness. Despite the fact that each species is prone to certain diseases, these are always relatively uncommon.
  • There are many other ways that pathogens that cause plant disease are indirectly transmitted, including by the wind, water, animals, insects, mites, nematodes, birds, and more.
  • Pathogens can be transferred from one diseased plant to another plant by birds flying through crops.
  • Wind can carry some plant harmful microorganisms a short way.
  • Raindrop splashing can spread certain bacteria, as seen in the instance of citrus canker disease.
  • The illnesses are spread by soil and agricultural activities as well because they cause dust to blow.

 

Question 2

In which of the following areas can GPS technology be used?

  1. Mobile phone operations

  2. Banking operations

  3. Controlling the power grids

Select the correct answer using the code given below:

(a) 1 only

(b) 2 and 3 only

(c) 1 and 3 only

(d) 1, 2 and 3

Ans: d

Sub-Theme: GPS Technology

  • A constellation of satellites transmitting navigational signals and a network of ground stations and satellite control stations used for monitoring and control make up the Global Positioning System (GPS), a space- based radio navigation system.
  • At the moment, 31 GPS satellites are in orbit around the Earth at a height of roughly 11,000 miles, giving users precise information on position, velocity, and time everywhere in the world and in any weather.
    • Every time and in practically any weather, GPS functions.
    • As a cooperative civil and military technical programme, the Global Positioning System— previously known as the Navstar Global Positioning System—began in 1973.
    • While global synchronised time was not the original purpose of GPS, it is today one of its most important features.
    • Cell tower synchronisation is required by telecom networks in order to pass calls between The clocks in the machinery that regulates current flow in overloaded networks are used by several electrical power grids.
    • In the Banking Sector ATM, credit card, and high-speed market transactions are timestamped using timing methods derived from GPS.
    • GPS clocks are used in everything from computer network synchronisation to digital television and radio broadcasting to seismic monitoring and weather reporting using Doppler radar.

 

Question 3

When the alarm of your smartphone rings in the morning, you wake up and tap it to stop the alarm which causes your geyser to be switched on automatically. The smart mirror in your bathroom shows the day’s weather and also indicates the level of water in your overhead After you take some groceries from your refrigerator for making breakfast, it recognises the shortage of stock in it and places an order for the supply of fresh grocery items. When you step out of your house and lock the door, all lights, fans, geysers and AC machines get switched off automatically. On your way to the office, your car warns you about traffic congestion ahead and suggests an alternative route, and if you are late for a meeting, it sends a message to your office accordingly.

In the context of emerging communication technologies, which one of the following terms best applies to the above scenario?

(a) Border Gateway Protocol

(b) Internet of Things

(c) Internet Protocol

(d) Virtual Private Network

Ans: b

Sub-Theme: AI Technology/New age or modern technology

The Correct answer is Option (b):

  • Internet of Things: The idea behind the Internet of Things (IoT) is to connect any device to the Internet and other connected devices as long as it has an on/off The Internet of Things (IoT) is a vast network of interconnected devices and people, all of which gather and exchange information about their environments and how they are used.
  • VPN: An encrypted connection between a device and a network via the Internet is known as a virtual private network, or Secure transmission of sensitive data is aided by the encrypted connection. It makes it impossible for unauthorised parties to eavesdrop on the traffic and enables remote work for the user. The use of VPN technology is common in business settings.
    • The Border Gateway Protocol (BGP): BGP enables data routing, is the protocol that powers the When a user in Singapore loads a website with origin servers in Argentina, BGP is the protocol that enables that communication to happen quickly and efficiently.
    • BGP is responsible for exploring all of the available paths that data could travel and picking the best route, which usually means hopping between autonomousThe Internet Protocol (IP): It is a protocol, or collection of guidelines, for addressing and routing data packets so they can move between networks and reach their intended The Internet divides data into smaller units known as packets for transmission. Each packet contains IP information, which enables routers to send packets to the appropriate location. Every device or domain that connects to the Internet is given an IP address, and data gets to the right place as packets are sent to the IP address associated with them.
  • NOTE: UPSC could have asked this question in a simple statement, but didn’t do that. The reason behind this long paragraph is to kill precious time and confuse the aspirants. Now if you’re aware and read about AI technology then you could easily pick up the keyword from the above paragraph such as ‘switched on/off automatically’, ‘smart mirror’, ‘traffic alert’, ’sending message automatically’, etc. and answer the question in much less time that too with confidence.

 

Question 4

Consider the following statements:

  1. Light is affected by

  2. The Universe is constantly

  3. Matter warps its surrounding space-time.

Which of the above is/are the prediction/ predictions of Albert Einstein’s General Theory of Relativity, often discussed in the media?

(a) 1 and 2 only

(b) 3 only

(c) 1 and 3 only

(d) 1, 2 and 3

Ans: d

Sub-Theme: Space science

  • General theory of relativity, is a geometric theory related to gravitation published by Albert Einstein in 1915 and currently defines gravitation in modern physics.
  • A coherent account of gravity as a geometric property of space and time, or four- dimensional spacetime, is provided by general relativity, which generalises special relativity and improves Newton’s law of universal gravitation.
  • In particular, the energy and momentum of all matter and radiation present have a direct relationship to the curvature of spacetime. The Einstein field equations, a set of partial differential equations, define the relationship.
  • Einstein’s work on special relativity has several ramifications, one of which is that time moves in relation to the Time dilation occurs when an object is in motion, which means that it perceives time more slowly while it is travelling quickly than when it is at rest.
  • General relativity is a theory of gravity where the fundamental idea is that instead of being an invisible force that attracts objects to one another, gravity is a curving or warping of space.
    • Statement 1 is correct: An item warps the space around it more dramatically the more large it is. Like a big ball lying on a rubber sheet, the sun, for instance, is large enough to bend space throughout our solar system. Because of this, Earth and the other planets orbit it in bent paths.
    • Statement 2 is correct: General Theory of Relativity showed that space and time are malleable or flexible, and ever expanding predicted that our universe was ever expanding. The theory of relativity states that massive objects distort spacetime around it. And since photons travel the shortest distance between two points, light appears to be bent when it travels through the distorted spacetime of a massive object.
    • Statement 3 is correct: An item warps the space around it more dramatically the more large it is. Like a big ball lying on a rubber sheet, the sun, for instance, is large enough to bend space throughout our solar system. Because of this, Earth and the other planets orbit it in bent paths.

 

Question 5

With reference to India’s satellite launch vehicles, consider the following statements:

  1. PSLVs launch the satellite useful for Earth resources monitoring whereas GSLVs are designed mainly to launch communication satellites.

  2. Satellites launched by PSLV appear to remain permanently fixed in the same position in the sky, as viewed from a particular location on Earth.

  3. GSLV Mk III is a four staged launch vehicle with the first and third stages using solid rocket motors; and the second and fourth stages using liquid rocket engines.

Which of the statements given above is/are correct?

(a) 1 only

(b) 2 and 3

(c) 1 and 2

(d) 3 only

Ans: a

Sub-Theme: Space technology

  • The satellite-launch vehicles (rockets) created by ISRO are the PSLV (Polar Satellite Launch Vehicle) and GSLV (Geosynchronous Satellite Launch Vehicle). While GSLVs are primarily made to launch communication satellites, PSLVs are meant to launch satellites helpful for monitoring Earth resources.
  • Geosynchronous Satellite Launch Vehicle Mark II (GSLV Mk II) is the largest launch vehicle created by India and currently in use. This three-stage launch vehicle of the fourth generation has four liquid strap- The third stage of the GSLV Mk II is the locally built and flight-tested cryogenic Upper Stage (CUS).
  • The GSLV Mk III is a three-stage launch vehicle featuring a solid fuel first stage, liquid fuel second stage, and cryogenic technology third stage using liquid hydrogen and liquid
  • India’s third-generation launch vehicle is the Polar Satellite Launch Vehicle (PSLV). It is the first launch vehicle made in India that has liquid With multiple successful missions, PSLV had established itself as India’s dependable and adaptable workhorse launch vehicle after its initial successful launch in October 1994.
  • Statement 1 is correct: PSLVs launch the satellites useful for Earth resources monitoring whereas GSLVs are designed mainly to launch communication satellites.
  • Statement 2 is incorrect: GSLV appears to remain permanently fixed in the same position in the sky.
  • Statement 3 is incorrect: GSLV Mk III is a three-stage launch vehicle with the first stage using solid fuel, the second stage liquid fuel and the third stage cryogenic technology with the use of liquid hydrogen and oxygen.
NOTE: In statement 2, “GSLV” has been replaced by “PSLV” to make statement incorrect and to increase confusion among PSLV and GSLV for aspirants. This is old technique of UPSC to replace one word by another, for instance – Augmented Reality (AR) and Virual Reality (VR). So elimination of statement 2 helps us to reach at 50:50. Stay focus, alert and Calm while reading question/statement.

 

Question 6

“3D printing” has applications in which of the following?

  1. Preparation of confectionery items

  2. Manufacture of bionic ears

  3. Automotive industry

  4. Reconstructive surgeries

  5. Data processing technologies

Select the correct answer using the code given below:

(a) 1, 3 and 4 only

(b) 2, 3 and 5 only

(c) 1 and 4 only

(d) 1, 2, 3, 4 and 5

Ans: d

Sub-Theme: Digital Technology/New age Technology

The Correct answer is Option (d): About 3D Printing:

  • 3D printing, also known as additive manufacturing, is a method of creating a three dimensional object layer-by-layer using a computer simulated design.
  • A 3D item is produced by the additive method of 3D printing, which involves building up layers of A final design is cut from a larger block of material in subtractive manufacturing techniques, which is the opposite of this. The result is minimal material waste due to 3D printing.
  • In the Netherlands, a 3-D printed food restaurant recently In America, a cell-seeded hydrogel matrix that was 3-D printed in the anatomical shape of a human ear was used to create a bionic ear. (Statements 1 and 2 are correct)
  • The automobile sector is one of the pioneers in the usage and integration of 3-D printing in Industries, with new routes being opened up at every level of the manufacture of motor vehicles, from the functional prototyping phases, design, tooling production, and components manufacturing. (Statement 3 is correct)
  • 3-D printing has also been used in reconstructive surgery to help repair damaged tissue like in cases of birth defects or traumatic injuries to severe burns or disease. (Statement 4 is correct)
  • 3-D printing is used to produce computer parts (such as circuit boards) which are used in processing of data. (Statement 5 is correct)

 

2017

 

Question 1

What is the application of Somatic Cell Nuclear Transfer Technology?

(a) Production of biolarvicides

(b) Manufacture of biodegradable plastics

(c) Reproductive cloning of animals

(d) Production of organisms free of diseases

Ans: c

Sub Theme: Application of Biotechnology

The most practical application of Somatic cell nuclear transfer (SCNT) is in the reproductive cloning of farm animals that have exceptional qualities, such as the ability to produce large quantities of milk.

Somatic Cell Nuclear Transfer Technology

  • The most practical application of Somatic cell nuclear transfer (SCNT) is in the reproductive cloning of farm animals that have exceptional qualities, such as the ability to produce large quantities of milk.
  • A somatic (body) cell’s nucleus is transferred to the cytoplasm of an enucleated egg(an egg that has had its own nucleus removed) using the somatic cell nuclear transfer (SCNT) procedure .
  • The SCNT-derived blastocyst used in reproductive cloning is implanted into the uterus of a surrogate mother, where it develops into a foetus and is carried to
  • The first mammal to be cloned via SCNT was a sheep named Dolly, who was born in 1996. The method might potentially be used to bring back extinct animals; for instance, frozen woolly mammoth cells could be employed as nuclear donors for enucleated elephant eggs.

biotechnology

 

Question 2. 

With reference to agriculture in India, how can the technique of ‘genome sequencing’, often seen in the news, be used in the immediate future?

  1. Genome sequencing can be used to identify genetic markers for disease resistance and drought tolerance in various crop plants.

2.This technique helps in reducing the time required to develop new varieties of crop plants.

3.It can be used to decipher the host pathogen relationships in crops.

Select the correct answer using the code given below:

(a) 1 only

(b) 2 and 3 only

(c) 1 and 3 only

(d) 1, 2 and 3

Ans: d

Sub-Theme: Genome sequencing

Genome Sequencing is figuring the sequence of DNA bases(nucleotides), such as As, Cs, Gs, and Ts that make up an organism’s DNA. These genetic letters total more than 3 billion and make up the human genome.

  • Statement 1 is correct: The identification of genetic markers for disease resistance and drought tolerance in different crop plants can be done via genome sequencing.
  • Statement 2 is correct: Genome sequencing is a method that will aid in genetic engineering and speed up the process of creating new crop plant kinds.
  • Statement 3 is correct: It also includes research on host-pathogen interactions in agricultural systems, like that of genes related to insecticide Resistance in Bactrocera dorsalis.

 

Question 3

Consider the following pairs:

Commonly used Unwanted or consumed materials  controversial  chemicals  likely  to  be found in them
1. Lipstick Lead
2. Soft drinks Brominated vegetable oils
3. Chinese fast Monosodium food glutamate

Which of the pairs given above is/are correctly matched?

(a) 1 only

(b) 2 and 3 only

(c) 1 and 3 only

(d) 1, 2 and 3

Ans: d

Sub-Theme: Harmful chemicals

  • Pair 1 is correct: Lip items like lipsticks and other cosmetics including eye shadow, blush, compact powder, shampoo, and body lotion frequently include the contaminant
  • Pair 2 is correct: In some sodas and soft drinks, the citrus flavour is sometimes prevented from breaking out by the food additive brominated vegetable oil. In Europe and Japan, it is prohibited as a food
  • Pair 3 is correct: A “non-essential” amino acid called glutamate, or glutamic acid salt, is what is known as monosodium glutamate (MSG). It transmits messages from one nerve cell to another as a neurotransmitter. It has “neuro-excitatory qualities,” which means it can stimulate neurons, because it is supposed to improve flavours. It is frequently utilised in fast food that is Chinese.

 

Question 4

Organic Light Emitting Diodes (OLEDs) are used to create digital displays in many devices. What are the advantages of OLED displays over Liquid Crystal displays?

  1. OLED displays can be fabricated on flexible plastic

  2. Roll-up displays embedded in clothing can be made using OLEDs.

  3. Transparent displays are possible using OLEDs.

Select the correct answer using the code given below:

(a) 1 and 3 only

(b) 2 only

(c) 1, 2 and 3

(d) None of the above statements is correct

Ans: c

Ans: c

Sub-Theme: Digital Technology

  • Statement 1 is correct: Regular OLEDs are usually fabricated on a glass substrate, but by replacing glass with a flexible plastic such as polyethylene terephthalate (PET) among others, OLEDs can be made both bendable and lightweight.
  • Statement 2 is correct: OLED displays can be fabricated on flexible plastic substrates leading to the possible fabrication of flexible organic light-emitting diodes for other new applications, such as roll-up displays embedded in fabrics or clothing.
  • Statement 3 is correct: LCD technology engages a backlight, whereas OLED has no backlighting function. Hence it can be used for making transparent displays.

 

2016

 

Question 1

In the context of the developments in Bioinformatics, the term ‘transcriptome’, sometimes seen in the news, refers to

(a) A range of enzymes used in genome editing

(b) The full range of mRNA molecules expressed by an organism

(c) The description of the mechanism of gene expression

(d) A mechanism of genetic mutations taking place in cells

Ans: b

Sub-Theme: Bioinformatics

Option (b) is correct: A transcriptome is the full range of messenger RNA, or mRNA, molecules expressed by an organism.

About Transcriptome:

  • A transcriptome is the full range of messenger RNA, or mRNA, molecules expressed by an organism.
  • The term “transcriptome” can also be used to describe the array of mRNA transcripts produced in a particular cell or tissue
  • In contrast with the genome, which is characterised by its stability, the transcriptome actively changes. In fact, an organism’s transcriptome varies depending on many factors, including the stage of development and environmental

 

Question 2

With reference to prepackaged items in India, is it mandatory to the manufacturer to put which of the following information on the main label, as per the Food Safety and Standards (Packaging and Labeling) Regulations, 2011?

  1. List of ingredients including additives
  2. Nutrition information
  3. Recommendation, if any, made by the medical profession about the possibility of any allergic reactions
  4. Vegetarian/nonvegetarian

Select the correct answer using the code given below.

(a) 1, 2 and 3

(b) 2, 3 and 4

(c) 1, 2 and 4

(d) 1 and 4 only

Ans: c

Sub-Theme: Food Safety Regulations

Option (c) is correct:

  • As per the Food Safety and Standards (Packaging and Labeling) Regulations, 2011, All pre-packaged items in India contain certain information such as Nutrition information, List of ingredients;, Vegetarian/Non-vegetarian/Vegan, Date of manufacture, Best before, Date of packing, Lot number, Code number, or Batch several-piece package, list of ingredients including additives, Food that has been prepackaged, a primary display panel, wholesale package.

 

Question 3

Which of the following statements is/are correct?

Virus can infect

  1. Bacteria

  2. Fungi

  3. Plants

Select the correct answer using the code given below

(a) 1 and 2 only

(b) 3 only

(c) 1 and 3 only

(d) 1, 2 and 3

Ans: d

Sub-Theme: Microorganisms

Option (d) is correct: All kinds of life, including animals, plants, and microbes like bacteria and fungi, are susceptible to virus infection.

  • Small infectious agents called viruses can only reproduce inside the live cells of other
  • All kinds of life, including animals, plants, and microbes like bacteria and fungi, are susceptible to virus infection.
  • The following four categories of host best describe viruses in terms of the sort of host they infect:
  1. Animal viruses, including human viruses, have DNA/RNA
  2. Plant viruses, including algal
  3. Mycoviruses (fungal viruses)-ds RNA
  4. Bacteriophages (bacterial viruses), including cyanophages-DN

 

Question 4

With reference to ‘LiFi’, recently in the news, which of the following statements is/are correct?

  1. It uses light as the medium for high-speed data transmission.

  2. It is a wireless technology and is several times faster than ‘WiFi’.

Select the correct answer using the code given below.

(a) 1 only

(b) 2 only

(c) Both 1 and 2

(d) Neither 1 nor 2

Ans: c

Sub-Theme: Communication Technology

LiFi

  • LiFi is a Visible Light Communications system that transmits wireless internet communications at very high speeds. This technology uses LED light bulb pulses of light that are undetectable to the human eye and within those emitted pulses, data can travel to and from receivers.
  • The receivers then gather data and translate the delivered information. This is essentially comparable to deciphering Morse code, but it happens millions of times each second, which is significantly faster. LiFi transmission rates can exceed 100 Gbps, which is 14 times faster than WiGig, also referred to as the fastest WiFi in the
  • In 2011, Harald Haas initially used the phrase in his Edinburgh TEDGlobal Li-Fi technology can transmit data and unlock capacity that is 10,000 times more than that which is available within the radio frequency by using the visible light spectrum.
  • The technology is similar to Wi-Fi in terms of its intended purpose, with the main technological distinction being that Wi-Fi uses radio frequency to induce a voltage in an antenna to transfer data, while Li-Fi employs modulation of light intensity to do
  • Statement 1 is correct: LiFi is a Visible Light Communications system transmitting wireless internet communications at very high speeds.
  • Statement 2 is correct: LiFi transmission speeds can go over 100 Gbps, 14 times faster than WiGig, also known as the world’s fastest WiFi.

 

Question 5

With reference to ‘Astrosat’,’ the astronomical observatory launched by India, which of the following statements is/are correct?

  1. Other than USA and Russia, India is the only country to have launched a similar observatory into space.

  2. Astrosat is a 2000 kg satellite placed in an orbit at 1650 km above the surface of the Earth.

Select the correct answer using the code given below.

(a) 1 only

(b) 2 only

(c) Both 1 and 2

(d) Neither 1 nor 2

Ans: d

Sub-Theme: Astronomical development

  • AstroSat is the first dedicated Indian astronomy mission aimed to study celestial sources in X-ray, optical and UV spectral bands. The payloads cover the energy bands of Ultraviolet (Near and Far), limited optical and X-ray regime (0.3 keV to 100keV).
  • The ability to simultaneously observe multiple celestial objects at multiple wavelengths with a single satellite is one of the special aspects of the AstroSat project.
  • India has joined the exclusive club of nations with its own space observatory with the successful launch of ASTROSAT, joining the US, Japan, Russia, and Europe. Along with six other foreign satellites, ASTROSAT, India’s first dedicated multi-wavelength space observatory, will aid in the knowledge of the cosmos.
  • Statement 1 is incorrect: ASTROSAT is India’s first dedicated multi-wavelength space observatory which will help in understanding the universe, and six other foreign satellites.
  • Statement 2 is incorrect: AstroSat with a lift- off mass of 1515 kg was launched on September 28, 2015 into a 650 km orbit inclined at an angle of 6 deg to the equator by PSLV-C30 from Satish Dhawan Space Centre, Sriharikota.
NOTE: it is an old technique of UPSC to weak with factual numbers. In the above question, the examiner changed the figures of ASTROSAT to make the statement incorrect. In 2016, the same was done with UNFCCC COP- 15 (Paris) in which the examiner replaced the pledge of mobilization of “USD100Bn” with “USD1000Bn”’ to make the statement incorrect. In 2017, Examiner replaced “six states” with “Five states’ ‘in “Western Ghats are spread over five states only” statement. They had also changed “12 states’ ‘ with “Five states” in “ Himalayas are spread over five States only” statement. However, the above -mentioned reasoning is applicable in most of the cases, but not in every case. It is advisable to stay alert whenever facts/data/ figures appear in a statement/question

 

Question 6

Consider the following statements:  The Mangalyaan launched by ISRO–

  1. Is also called the Mars Orbiter Mission
  2. Made India the second country to have a spacecraft orbit the Mars after USA
  3. Made India the only country to be successful in making its spacecraft orbit the Mars in its very first attempt

Which of the statements given above is/are correct?

(a) 1 only

(b) 2 and 3 only

(c) 1 and 3 only

(d) 1, 2 and 3

Ans: c

Sub-Theme: Space mission

Mangalyaan/Mars Orbiter Mission.

  • Mars Orbiter Mission (MOM), the maiden interplanetary mission of ISRO, that was launched on November 5, 2013 on PSLV-C25 entered into Martian orbit on September 24, 2014 in its first attempt.
  • MOM has completed 1000 Earth days in its orbit, on June 19, 2017, well beyond its designed mission life of six months. 1000 Earth days corresponds to 24 Mars Sols (Martian Solar day) and MOM completed 388 orbits.
  • It is India’s first interplanetary mission, and ISRO has joined the Soviet space programme, NASA, and the European Space Agency as the fourth space agency to reach Mars. It is the first country in Asia to enter Mars’ orbit, as well as the first country ever to do so.
  • The world’s most cost-effective inter- planetary expedition cleared the door for affordable access to outer space. With a budget of Rs 450 Cr, the launch vehicle, spacecraft, and ground segment were all
  • Statement 1 is correct: The Mangalyaan launched by ISRO is also called the Mars Orbiter Mission.
  • Statement 2 is incorrect: It is India’s first interplanetary mission and ISRO has become the fourth space agency to reach Mars, after the Soviet space Programme, NASA and the European Space Agency.
  • Statement 3 is correct: It is the first Asian nation to reach Mars orbit, and the first nation in the world to do so in its first attempt.

 

Question 7

India is an important member of the ‘International Thermonuclear Experimental Reactor’. If this experiment succeeds, what is the immediate advantage for India?

(a) It can use thorium in place of uranium for power generation

(b) It attain a global role in satellite-navigation

(c) It can drastically improve the efficiency of its fission reactors in power generation

(d) It can build fusion reactors for power

Ans: d

Sub-Theme: Nuclear Energy

The Correct answer is Option (d):

About International Thermonuclear Experimental Reactor:

  • A worldwide nuclear fusion research and engineering megaproject called the International Thermonuclear Experimental Reactor (ITER) aims to mimic the fusion processes of the Sun to produce energy on Earth.
  • The Sun and other stars derive their energy from fusion. Hydrogen nuclei clash, fuse into heavier helium atoms, and unleash enormous amounts of energy in the intense heat and gravity at the Centre of these star
  • An experimental device created to capture fusion energy is called a tokamak. The energy generated by the fusing of atoms inside a tokamak is absorbed as heat by the vessel’s walls.
  • Over 35 countries are working together on the ITER Project. China, the European Union, India, Japan, Korea, Russia, and the United States, who are all ITER members, have pooled their resources to break through one of science’s greatest barriers: replicating on Earth the limitless energy that powers the Sun and the stars.
  • ITER (International Thermonuclear Experimental Reactor) marked the beginning of the ITER Tokamak Assembly in July 2020 in Saint-Paul-Lez-Durance, France.

 

2015

 

Question 1

With reference to ‘Near Field Communication (NFC) Technology’, which of the following statements is/ are correct?

  1. It is a contactless communication technology that uses electromagnetic radio fields.

  2. NFC is designed for use by devices which can be at a distance of even a metre from each other.

  3. NFC can use encryption when sending sensitive

Select the correct answer using the codes given below.

(a) 1 and 2 only

(b) 3 only

(c) 1 and 3 only

(d) 1, 2 and 3

Ans: c

Sub-Theme: Communication Technology

  • Near Field Communication (NFC) is a contactless communication technology based on a radio frequency (RF) field using a base frequency of 13.56 MHz. NFC technology is designed to exchange data between two devices simply through a touch gesture.
  • In comparison to other wireless technologies, which have a working distance of several metres, NFC technology is designed for an operation distance of a few centimetres, making it more challenging for attackers to record the communication between an NFC Forum Device and an NFC Forum Tag.
  • Additionally, the NFC Forum Device user chooses through a touch gesture with which entity the NFC connection should occur, making it more challenging for the attacker to connect.
  • As a result, compared to other wireless communication protocols, the security level of NFC communication is by design higher.
  • Statement 1 is correct: NFC technology is perfectly designed to exchange data between two devices through a simple touch gesture.
  • Statement 2 is incorrect: NFC technology is designed for an operation distance of a few centimeters, this makes it more difficult for attackers to record the communication between an NFC Forum Device and an NFC Forum Tag compared to other wireless technologies which have a working distance of several meters.
  • Statement 3 is correct: The security level of the NFC communication is by default higher compared to other wireless communication protocols.

 

Question 2

With reference to the use of nanotechnology in the health sector, which of the following statements is/are correct?

  1. Targeted drug delivery is made possible by nanotechnology.

  2. Nanotechnology can largely contribute to gene therapy.

Select the correct answer using the codes given below.

(a) 1 only

(b) 2 only

(c) Both 1 and 2

(d) Neither 1 nor 2

Ans: c

Sub-Theme: Nanotechnology

  • Nanotechnology deals with the understanding and controlling of matter at level of 1 and 100 nanometers, where unique phenomena enable novel
  • Targeted drug distribution by nanoscale drug carriers is one of the fundamental tenets of nanomedicine. Numerous nanoscale drug delivery technologies, such as liposomes, dendrimers, quantum dots, nanotubes, polymeric biodegradable nanoparticles, and nanocapsules, have been studied.
  • In addition to enabling new therapeutic classes and encouraging the re-investigation of previously unfeasible new molecular entities that are pharmaceutically suboptimal but   biologically active, nanoscale delivery vehicles can improve the therapeutic efficacy and minimise side effects associated with existing medications.
  • The development of medical nanotechnology can significantly advance genetic treatment. If diseases are handled from a genetic perspective, they are simple to treat. Therefore, using nanotechnology will enable medical professionals to treat illnesses by focusing on their fundamental causes rather than just their symptoms.
  • Statement 1 is correct: One of the key aspects of nanomedicine is targeted drug delivery by nanoscale drug carriers.
  • Statement 2 is correct: Medical nanotechnology can largely contribute to genetic therapy and improvement.

 

Question 3

In the context of modern scientific research, consider the following statements about ‘IceCube’, a particle detector located at South Pole, which was recently in the news:
1. It is the world’s largest neutrino detector, encompassing a cubic kilometer of ice.
2. It is a powerful telescope to search for dark matter.
3. It is buried deep in the ice.

Which of the statements given above is/are correct?

(a) 1 only

(b) 2 and 3 only

(c) 1 and 3 only

(d) 1, 2 and 3

Ans: d

Sub-Theme: Space observatory

  • The Ice Cube Neutrino Observatory is the first detector of this type, designed for observing the cosmos from deep within the South Pole ice. An international group of scientists responsible for the scientific research makes up the IceCube Collaboration.
  • The most violent astrophysical sources, including as stellar explosions, gamma- ray bursts, and catastrophic occurrences involving black holes and neutron stars, can all be studied using the information provided by these high-energy cosmic
  • Statement 1 is correct: With a cubic kilometre of ice in its footprint, Ice Cube is the largest neutrino detector ever built. It is buried up to a depth of 2.5 km.
  • Statement 2 is correct: The Ice Cube telescope is an effective tool for looking for dark matter and may shed light on the physics mechanisms underlying the mysterious origin of the highest energy particles in nature.
  • Statement 3 is correct: Ice Cube, which encompasses a cubic kilometre of ice, looks for neutrinos, which are nearly massless subatomic particles.

 

Question 4

The term ‘Goldilocks Zone’ is often seen in the news in the context of

(a) The limits of habitable zone above the surface of the Earth

(b) Regions inside the Earth where shale gas is available

(c) Search for the Earth-like planets in outer space

(d) Search for meteorites containing precious metals

Ans: c

Sub-Theme: Space Science

The Correct answer is Option (c):

  • The Goldilocks Zone is the habitable zone around a star where the temperature is optimum – not too hot and not too cold – for liquid water to exist on a planet.
  • It is a metaphor of the children’s story “Goldilocks and the Three Bears,” in which a young girl selects from sets of three objects, eschewing the extremes (such as those that are enormous or small, hot or cold, ), and deciding on the one that is “just right” in the middle.
  • The Goldilocks zone of the Sun surrounds the Earth. All of Earth’s water would freeze if it were located where the dwarf planet Pluto is; however, all of Earth’s water would boil out if it were located where Mercury
  • Water is a fundamental component for life as we know it and is where life first began on Earth. So any rocky exoplanet in the habitable zone will be considered by researchers as they look for evidence of alien life.

 

Question 5

With reference to ‘fly ash’ produced by the power plants using coal as fuel, which of the following statements is/are correct? 

  1. Fly ash can be used in the production of bricks for building construction.

  2. Fly ash can be used as a replacement for some of the Portland cement contents of concrete.

  3. Fly ash is made up of silicon dioxide and calcium oxide only, and does not contain any toxic

Select the correct answer using the codes given below.

(a) 1 and 2

(b) 2 only

(c) 1 and 3

(d) 3 only

Ans: a

Sub-Theme: Environmental Chemistry

About Fly Ash:

  • Fly ash is a fine grey powder that consists mostly of spherical, glassy particles that are produced as a byproduct in coal-fired power
  • Pozzolanic characteristics in fly ash allow it to interact with lime to produce cementitious compounds. It is frequently referred to as an additional cementitious
  • Fly ash is a by product of combustion that is made up of the small particles that rise with the flue gases. Bottom ash is ash that does not Fly ash is typically used in an industrial setting to describe the ash created during coal burning.
  • The main chemical components present in fly ash are:
    • Silicon dioxide
    • Aluminium oxide
    • Ferric oxide
    • Calcium oxide (occasionally) (Statement 3 is incorrect)
  • Fly ash particle condenses while suspended in exhaust gases, and electrostatic precipitators or filter bags are used to remove it. Fly ash particles are typically spherical in shape and range in size from 0.5 µm to 300 µm because they harden quickly when suspended in the exhaust fumes.
  • Fly ash has numerous advantages in both cement and non-cement applications and considerably enhances the performance of concrete. Additionally, fly ash looks to work well as a catalyst for the high-temperature pyrolysis process, which transforms polyethylene into a material akin to crude oil, when treated with sodium hydroxide. (Statements 1 & 2 are correct)

 

Question 6

With reference to ‘fuel cells’ in which hydrogen-rich fuel and oxygen are used to generate electricity, consider the following statements: 

  1. If pure hydrogen is used as a fuel, the fuel cell emits heat and water as by-products.

  2. Fuel cells can be used for powering buildings and not for small devices like laptop computers.

  3. Fuel cells produce electricity in the form of Alternating Current (AC).

Which of the statements given above is/are correct?

(a) 1 only

(b) 2 and 3 only

(c) 1 and 3 only

(d) 1, 2 and 3

Ans: a

Sub-Theme: Non-Conventional Energy/ Renewable Energy

Statement 1 is correct: A fuel cell generates electricity in a clean and effective manner using the chemical energy of hydrogen or other fuels. Electricity, water, and heat are the only by products if hydrogen is the fuel.

Statement 2 is incorrect: In terms of the diversity of potential applications, fuel cells are exceptional; they can run on a variety of fuels and feedstocks and can power devices as big as utility power plants and as tiny as laptop computers.

Statement 3 is incorrect: Fuel cells produce electricity in the form of direct current (DC). Electrons can only move in one direction in a DC circuit. Alternating current (AC), which circulates in both directions on alternating cycles, provides the electricity in our homes and workplaces.

 

2014

 

Question 1

Consider the following pairs:

Vitamin Deficiency :    Disease

  1. Vitamin C                   Scurvy

  2. Vitamin D                   Rickets

  3. Vitamin E                   Night blindness

Which of the pairs given above is/are correctly matched?

(a) 1 and 2 only

(b) 3 only

(c) 1, 2 and 3

(d) None

Ans: a

Sub-Theme: Vitamins

    • In human bodies, deficiencies in one or more nutrients can lead to illnesses or problems. The diseases are illnesses that develop over an extended period of time as a result of nutrient deficiency
    • Vitamin A  Night blindness
    • Vitamin B1  Beriberi
    • Vitamin B2  Ariboflavinosis
    • Vitamin B3  Pellagra
    • Vitamin B5  Paresthesia
    • Vitamin B6  Anemia
    • Vitamin B7  Dermatitis, enteritis
    • Vitamin B9 & Vitamin B12  Megaloblastic anemia
    • Vitamin C ___________ Scurvy, Swelling of

Gums

  • Vitamin D  Rickets & Osteomalacia
  • Vitamin E_________ Less Fertility
  • Vitamin K ________  Non-Clotting of Blood

 

Question 2

Consider the following diseases:

  1. Diphtheria

  2. Chickenpox

  3. Smallpox

Which of the above diseases has/have been eradicated in India ?

(a) 1 and 2 only

(b) 3 only

(c) 1, 2 and 3

(d) None

Ans: b

Sub-Theme: Disease

    • Diphtheria: The bacteria Corynebacterium diphtheria is the cause of the infectious illness diphtheria. The throat and upper airways are the site of the main infection. According to data from the Central Bureau of Health Intelligence, India reported 41,672 cases of diphtheria between 2005 and 2014 (an average of 4,167 cases per year), with 897 deaths (a case fatality ratio of 2.2%). 84% of the cases recorded nationwide were in ten states: Andhra Pradesh, Assam, Delhi, Gujarat, Haryana, Karnataka, Nagaland, Maharashtra, Rajasthan, and West Bengal.
    • Chickenpox: The varicella-zoster virus, which causes chickenpox, is a highly contagious   While  it  primarily affects children, it can also impact adults. People are still concerned about chicken pox, and the government is working to eradicate it completely.
      • Smallpox: Smallpox is an acute contagious illness brought on by the variola virus, an orthopoxvirus. Before it was destroyed, it was one of the most terrible diseases that had ever struck mankind. It has supposedly been.

 

Question 3

In addition to fingerprint scanning, which of the following can be used in the biometric identification of a person?

  1. Iris scanning

  2. Retinal scanning

  3. Voice recognition

Select the correct answer using the code given below.

(a) 1 only

(b) 2 and 3 only

(c) 1 and 3 only

(d)1, 2 and 3

Ans: d

Sub-Theme: Biometric

  • In computer science Biometrics identification is used as a form of identification and  access control
  • Biometric identifiers are the distinct, measurable characteristics used to label individuals.
  • Behavioural Indicators: Physiological characteristics are distinguished from behavioural traits when classifying biometric identifiers. Fingerprint, palm veins, face recognition, DNA, palm print, hand geometry, iris recognition, retina, and odour/scent are examples of physiological biometric identifiers. Gait, voice, and typing rhythm are behavioural indicators.
  • Iris scanning
    • It is a form of biometric technology which uses visible and near-infrared light to take a high-contrast photograph of a person’s iris.
    • Retinal scanning
    • It maps the unique patterns of a person’s retina.
    • The blood vessels within the retina absorb light more readily than the surrounding tissue and thus easily identify with appropriate lighting.
    • It is highly accurate and difficult to spoof, in terms of identification.
  • Voice recognition
    • It is a computer software program which can decode the human voice.
    • It is used to operate a device or perform commands

 

 

Question 4

With reference to Agni-IV Missile, which of the following statements is/are correct?

  1. It is a surface-to-surface

  2. It is fuelled by liquid propellant

  3. It can deliver one-tonne nuclear warheads about 7500 km away.

Select the correct answer using the code given below.

(a) 1 only

(b) 2 and 3 only

(c) 1 and 3 only

(d) 1, 2 and 3

Ans: a

Sub-Theme: Defence Technology

About Agni-IV Missile:

  • It is a nuclear-capable long-range ballistic missile with a strike range of 4,000 km. (Statement 3 is incorrect:)
  • It is a two-stage solid propellant missile. (Statement 2 is incorrect)
  • It is a surface-to-surface  (Statement 1 is correct)
  • It is 20 metres long with a weight of 17 tonnes.
  • It is the fourth in the Agni series of missiles.
  • It can deliver one-tonne nuclear warheads.
Agni Class of Missiles:

  • They are the mainstay of India’s nuclear launch capability.
  • Range of other Agni Missiles:
    • Agni I: Range of 700-800 km.
    • Agni II: Range more than 2000 km.
    • Agni III: Range of more than 2,500 Km
    • Agni IV: Range is more than 3,500 km and can fire from a road mobile launcher.
    • Agni-V: The longest of the Agni series, an Inter-Continental Ballistic Missile (ICBM) with a range of over 5,000 km

 

IGMDP (Integrated Guided Missile Development Program)

  • It was conceived by A.P.J. Abdul Kalam to enable India attain self-sufficiency in the field of missile technology. It was approved by the Government of India in 1983 and completed in March 2012.
  • The 5 missiles (P-A-T-N-A) developed under this program are:
    • Prithvi: Short range surface to surface ballistic missile.
    • Agni: Ballistic missiles with different ranges, i.e. Agni (1,2,3,4,5)
    • Trishul: Short range low level surface to air missile.
    • Nag: 3rd generation anti-tank missile.
    • Akash: Medium range surface to air missile.

 

Question 5

Which one of the following is the process involved in photosynthesis?

(a) Potential energy is released to form free energy.

(b) Free energy is converted into potential energy and stored.

(c) Food is oxidised to release carbon dioxide and water.

(d) Oxygen is taken, and carbon dioxide and water vapour are given out.

Ans: b

Sub-Theme: Photosynthesis

About Photosynthesis:

  • Photosynthesis– It is the process through which green plants and some other organisms convert light energy into chemical
  • The functions of the chlorophyll, a green pigment, are related to the transformation of inefficient solar energy into useful chemical energy. The majority of the time, the photosynthetic process makes use of water and emits the oxygen that is vitally necessary for our continued existence.

6H2O + 6CO2 ¾¾→ C6H12O6+ 6O2

  • Photosynthesis uses light energy for conversion of carbon dioxide and water into glucose and oxygen gas.
  • The potential energy stored in the molecular bonds of glucose convert into kinetic energy after cellular respiration.
  • Chemical bonds are a type of stored or “potential” energy because when the bonds break, energy is released.
  • Photosynthesis converts water and carbon dioxide into sugar and oxygen through sunlight as the A by product of the photosynthesis process is the released oxygen into the atmosphere.

 

Question 6

Which of the following statements is/are correct regarding vegetative propagation of plants?

  1. Vegetative propagation produces a clonal population.

  2. Vegetative propagation helps in eliminating the virus.

  3. Vegetative propagation can be practised most of the year.

Select the correct answer using the code given below.

(a) 1 only

(b) 2 and 3 only

(c) 1 and 3 only

(d) 1, 2 and 3

Ans: c

Sub-Theme: Plant Reproduction

  • Statement 1 is correct: Vegetative reproduction is a type of asexual reproduction in plants. In this process new organisms arise without production of seeds . It helps in clone development.
  • Statement 2 is incorrect: Vegetative reproduction only includes mitosis, which guarantees that the genetic information in the DNA of vegetative progeny (child) is identical to that in the mother plant and allows for year-round use. However, it has no effect on the removal of viruses. A plant that has been repeatedly infected by a virus typically carries the infection for its whole lifespan. Therefore, any vegetative portions used for reproduction stay contaminated.
  • Statement 3 is correct: Vegetative propagation also allows plants to circumvent the immature seedling phase and reach the mature phase faster. In nature, that increases the chances for a plant to successfully reach maturity, and, commercially, it saves farmers a lot of time and money as it allows for faster crop overturn. It can be practised throughout the year.

 

Question 7

Which of the following is/are the example/examples of chemical change?

  1. Crystallisation of sodium chloride

  2. Melting of ice

  3. Souring of milk

Select the correct answer using the code given below.

(a) 1 and 2 only

(b) 3 only

(c) 1, 2 and 3

(d) None

Ans: b

Sub-Theme: Basic and Everyday Science

The Correct answer is Option (b): About Chemical Change:

  • It involves  the  transformation  of  one material into another, the creation of new materials with unique properties, and the creation of one or more new substances.
  • It happens when two compounds mix to create a new material (synthesis or either decomposes to form more substances). An example of a chemical change that results in a reaction is the oxidation reaction. Examples include burning wood, sour milk, cooking eggs, heating sugar to make caramel, and rusting iron.

About Physical Change:

  • No new chemical species
  • Since the identity of the matter does not change, changing a pure substance’s state from solid to liquid or gas constitutes a physical change.For instance: Ice melting, boiling water, sodium chloride crystallisation, etc.

 

Question 8

Consider the following techniques/phenomena:

  1. Budding and grafting in fruit plants

  2. Cytoplasmic male sterility

  3. Gene silencing

Which of the above is/are used to create transgenic crops ?

(a) 1 only

(b) 2 and 3 only

(c) 1 and 3 only

(d) None

Ans: b

Sub-Theme: Genetics/Biotechnology

  • Statement 1 is incorrect: In the budding process, a bud is taken from one plant and grown on another, it does not lead to any genetic modification.
  • Statement 2 is correct: The Cytoplasmic male sterility systems are widely utilised in crop plants for hybrid breeding due to the convenience of controlling sterility.
  • Statement 3 is correct: Gene Silencing using RNA interference technology, transcriptional gene silencing, virus induced gene silencing has been used in horticultural techniques

 

Question 9

Consider the following statements:

  1. Maize can be used for the production of starch.

  2. Oil extracted from maize can be a feedstock for biodiesel.

  3. Alcoholic beverages can be produced by using maize.

Which of the statements given above is/are correct?

(a) 1 only

(b) 1 and 2 only

(c) 2 and 3 only

(d) 1, 2 and 3

Ans: d

Sub-Theme: Basic and Everyday science

  • Statement 1 is correct: Corn wet-milling is a process of breaking corn kernels into their component parts: corn oil, protein, corn starch, and fibre. It uses water and a series of steps to separate the parts to be used for various products.
  • Statement 2 is correct: Chemically speaking, biodiesel is a combination of long-chain fatty acid methyl esters (FAMEs). Maize oil can be used to make ethanol or biodiesel. The sort of alcohol found in the alcoholic beverages we consume is a substance known as ethanol.
  • Statement 3 is correct: Grain, fruit, or vegetable fermentation is the process used to produce alcohol. Grain such as maize and other grains is a typical source of ethanol.

 

Question 10

Among the following organisms, which one does not belong to the class of the other three?

(a) Crab

(b) Mite

(c) Scorpion

(d) Spider

Ans: a

Sub-Theme: Species and Subspecies

The Correct answer is Option (a): About Organisms:

  • Mite, Spider and Scorpion belong to the category of Arachnids, whereas Crabs belong to the category of crustaceans.
  • Arachnida is a class of joint-legged invertebrate animals (arthropods), in the subphylum Arachnida includes, among others, spiders, scorpions, ticks, mites, pseudoscorpions, harvestmen, camel spiders, whip spiders and vinegaroons.
  • Nearly all arachnids are terrestrial and have eight legs, while some species have evolved their front pair of legs into sensory organs.
  • The group of arthropods known as crustaceans, which includes crabs, lobsters, crayfish, shrimp, krill, barnacles, brine shrimp, copepods, ostracods, and mantis shrimp, is quite big.
  • The majority of crustaceans are aquatic, free-living creatures, although others are sessile, terrestrial, or parasitic (e.g., woodlice, fish lice, or tongue worms) (e.g. barnacles).

 

Question 11

Which one of the following is the correct sequence of a food chain?

(a) Diatoms-Crustaceans-Herrings

(b) Crustaceans-Diatoms-Herrings

(c) Diatoms-Herrings-Crustaceans

(d) Crustaceans-Herrings-Diatoms

Ans: a

Sub-Theme: Food Chain

The Correct answer is Option (a): About Food Chain:

  • Food chain is the pathway for transfer of food energy from green plants (producers) through a series of organisms with repeated consumption and being consumed.
  • Eg. Grasses → Grasshopper → Frog → Snake → Hawk/Eagle.
  • Every step in the food chain is called trophic level. A food chain starts with producers and ends with top The trophic level of an organism is the position at which it is placed in a food chain. The food chain starts with a producer and ends with a top consumer.
  • Phytoplankton are the primary producers in the oceans. They include:
    • diatoms (unicellular algae),
    • coccolithophores (unicellular, eukaryotic protist),
    • Cyanobacteria (Blue Green algae)– Synechococcus, Prochlorococcus, Nostoc, spirogyra etc.
    • Dinoflagellates (flagellated protists).
  • The group of arthropods known as crustaceans, which includes crabs, lobsters, crayfish, shrimp, krill, and barnacles, is quite big.
  • Fish called herrings devour

 

Question 12

With reference to technology for solar power production, consider the following statements:

  1. ‘Photovoltaics’ is a technology that generates electricity by direct conversion of light into electricity, while ‘Solar Thermal’ is a technology that utilises the Sun’s rays to generate heat which is further used in the electricity generation process.

  2. Photovoltaics-generates Alternating Current (AC), while Solar Thermal generates Direct Current (DC).

  3. India has a manufacturing base for Solar Thermal technology, but not for photovoltaics.

Which of the statements given above is/are correct?

(a) 1 only

(b) 2 and 3 only

(c) 1, 2 and 3

(d) None

Ans: a

Sub -Theme: Non-conventional Energy/ Renewable Energy/Solar Energy

  • Statement 1 is correct: Photovoltaic (PV) technologies – which are commonly known as solar panels – produce power through devices that absorb energy from sunlight and convert it into electrical energy through semiconducting materials. Then, these components—known as solar cells—are coupled to create bigger power-generating structures called modules or panels. However, in solar thermal, a fluid is boiled using sunshine. Depending on the use, this liquid could be water or any other liquid.
  • Statement 2 is incorrect: Solar thermal does not generate electricity, it does convertion of solar radiation into heat.
  • Statement 3 is incorrect: India has a manufacturing base for both solar thermal and photovoltaics.

 

Question 13

Consider the following:

  1. Bats

  2. Bears

  3. Rodents

The phenomenon of hibernation can be observed in which of the above kinds of animals?

(a) 1 and 2 only

(b) 2 only

(c) 1, 2 and 3 only

(d) Hibernation cannot be observed in any of the above

Ans: c

Sub-Theme: Adaptation/Response to abiotic factors

The Correct answer is Option (c): About Hibernation:

  • Hibernation is a state of inactivity and metabolic depression in endotherms. It has a low metabolic rate, slow breathing, slow heartbeat, and low body temperature. For example, it is present in bats, bears, and
  • When there is not enough food available, hibernation serves to conserve An endothermic animal lowers its metabolic rate and subsequently its body temperature to achieve this energy savings.
  • Larger species develop hyperphagia, when they consume a lot of food and store the energy as fat.
  • In many tiny species, food storage takes the place of eating and fat development.
  • An endotherm is an organism which maintains its body at a metabolically stable temperature, by the use of heat released by its internal bodily functions rather than relying almost purely on ambient heat e.g People, polar bears, penguins, and prairie

 

Question 14

Which of the following adds/add carbon dioxide to the carbon cycle on the planet Earth?

  1. Volcanic action

  2. Respiration

  3. Photosynthesis

  4. Decay of organic matter

Select the correct answer using the code given below:

(a) 1 and 3 only

(b) 2 only

(c) 1, 2 and 4 only

(d) 1, 2, 3 and 4

Ans: c

Sub-Theme: Carbon cycle

  • Statement 1 is correct: Volcanic action leads to the release of Carbon dioxide.
  • Statement 2 is correct: Respiration is the movement of oxygen from the outside environment to the cells within tissues and the release of carbon dioxide in the opposite direction.
  • Statement 3 is incorrect: The photosynthesis process absorbs carbon dioxide and emits oxygen. Therefore, photosynthesis does not cause the carbon cycle to gain more carbon dioxide. The process by which plants convert carbon dioxide, water, and sunshine into oxygen and sugar-based energy is known as photosynthesis.
  • Statement 4 is correct: The process of decomposition releases Carbon dioxide.

 

Question 15

Lichens, which are capable of initiating ecological succession even on a bare rock, are actually a symbiotic association of

(a) Algae and bacteria

(b) Algae and fungi

(c) Bacteria and fungi

(d) Fungi and mosses

Ans: b

Sub-Theme: Ecological succession

The Correct answer is Option (b): About Ecological Succession:

  • Ecological succession  is  the  process of change in the species structure of an ecological community over time. Successional dynamics beginning with colonisation of an area that has not been previously occupied by an ecological community are referred to as primary succession. This includes newly exposed rock or sand surfaces, lava flows, and newly exposed glacial tills. The stages of primary succession include pioneer microorganisms, plants (lichens and mosses), grassy stage, smaller shrubs, and trees.
  • Lichens are a complex life form that is a symbiotic partnership of two separate organisms, a fungus and an alga where the dominant partner is the fungus, that provides the lichen the majority of its characteristics, from its thallus shape to its fruiting bodies.

 

2013

 

Question 1

Mycorrhizal biotechnology has been used in rehabilitating degraded sites because mycorrhiza enables the plants to:

  1. Resist drought and increase absorptive area

  2. Tolerate extremes of pH

  3. Resist disease infestation

Select the correct answer using the codes given below:

(a) 1 only

(b) 2 and 3 only

(c) 1 and 3 only

(d) 1, 2 and 3

Ans: d

Sub-Theme: Mycorrhizal biotechnology

  • Statement  1  is   correct : Trees host mycorrhizal fungus in their roots so they can absorb nutrients from the ground and resist the drought condition. These fungi exchange the carbon from the tree for vital nutrients including nitrogen, phosphate, and potassium from the soil.
  • Statement 2 is correct: Mycorrhizal plants can withstand unfavourable soil conditions, high pH levels, droughts, and poisonous heavy metals.
  • Statement 3 is correct: Mycorrhizae are symbiotic relationships between plant roots and specific soil fungi that are important for the ecosystem’s nutrient cycling and that help shield plants from environmental and cultural stress. Additionally, mycorrhizal fungi’s synthesis of oxalic acid contributes to disease suppression.

 

Question 2

Recombination DNA technology (Genetic Engineering) allows genes to be transferred:

  1. Across different species of plants

  2. From animals to plants

  3. From microorganisms to higher organisms

Select the correct answer using the codes given below.

(a) 1 only

(b) 2 and 3 only

(c) 1 and 3 only

(d) 1, 2 and 3

Ans: d

Sub-Theme: DNA technology/Genetic Engineering

  • Recombinant DNA technology entails changing genetic material outside of an organism to produce living organisms or their products with improved and desired
  • Using the appropriate vectors, this approach entails inserting DNA fragments from a number of sources that have the desired gene sequence.
  • Manipulation in an organism’s genome is carried out either through the introduction of one or several new genes and regulatory elements or by decreasing or blocking the expression of endogenous genes through recombining genes and elements.
  • It allows genes to be transferred across different species of plants, animals to plants and from microorganisms to higher organisms.

 

Question 3

Consider the following minerals:

  1. Calcium

  2. Iron

  3. Sodium

Which of the minerals given above is/are required by the human body for the contraction of muscles?

(a) 1 only

(b) 2 and 3 only

(c) 1 and 3 only

(d) 1, 2 and 3

Ans: d

Sub-Theme: Components of Food/Balanced Diet

Option (d) is correct: Specific minerals like calcium, Iron sand sodium play a part in muscle contraction.

  • Skeletal muscle attached to our skeleton, contracts to maintain our posture and support movement, whereas cardiac muscle makes up our heart and contracts with each heartbeat.
  • Our airways, digestive system, blood vessels, and other tissues are lined with smooth muscle, and the contraction of this muscle is important for digestion, circulation, and other physiological activities.
  • Specific minerals play a part in muscle contraction:
    • Calcium: Calcium is essential for activating enzymes that cause muscle contraction.
    • Iron: Iron helps to support continued functioning of the muscle by supporting energy production in your muscle cells, so that muscle fibres always have the energy they need to contract properly. Also supports healthy circulation so that muscles can receive additional oxygen from the bloodstream and helps muscles store oxygen to power muscle contractions.
    • Sodium: Sodium balances potassium to maintain membrane potential, it is necessary for muscular contraction.
NOTE: This is a simple, easy and direct question from NCERT. Therefore, it is very much important for the aspirants to read and understand each and every concept including the minor details from the NCERTs.

 

Question 4

Which of the following statements is/are correct?

  1. Viruses lack enzymes necessary for the generation of energy

  2. Viruses can be cultured in any synthetic medium.

  3. Viruses are transmitted from one organism to another by biological vectors only

Select the correct answer using the codes given below.

(a)  1 only

(b)  2 and 3 only

(c) 1 and 3 only

(d)  1, 2 and 3

Ans: a

Sub-Theme: Virus

  • Statement 1 is correct: Viruses do not have the chemical machinery needed to survive on their own.
  • Statement 2 is incorrect: The virus can’t reproduce itself without another cell to “build” its copies. Therefore we can’t grow a virus on synthetic media.
  • Statement 3 is incorrect: Viruses enter the body from the environment or other individuals from soil to water to air via nose, mouth, or any breaks in the skin and seek a cell to infect.
  • Viruses lack the necessary chemical apparatus for independent survival.
  • Viruses look for host cells where they can reproduce. On artificial medium, such as agar, viruses cannot thrive, although they can on bacterial On the other hand, viruses resemble little aliens.
  • Since they don’t consume nutrients or excrete waste, they aren’t really considered to be “living.” Instead, they attach to another cell (such as a bacterium) and inject it with their own viral genetic material.
  • Without a second cell to “build” its copies in, the virus is unable to replicate. As a result, we are unable to grow viruses on artificial media.
  • Viral contamination from the environment or other people enters the body through the nose, mouth, or any openings in the air, water, or soil.

 

Question 5  

Which of the following diseases can be transmitted from one person to another through tattooing?

  1. Chikungunya

  2. Hepatitis B

  3. HIV-AIDS

Select the correct answer using the codes given below:

(a) 1 only

(b) 2 and 3 only

(c) 1 and 3 only

(d) 1, 2 and 3

Ans: b

Sub-Theme: Disease transmission

Option (b) is correct:

  • Tattoos may cause diseases because tattoo instruments come in contact with blood and bodily fluids if the instruments are used on more than one person without being sterilised.
  • When tattoo instruments are used on multiple people without being sanitised, they come into contact with blood and body fluids and may spread illnesses.
  • However, tattoo-related infections are uncommon in sterile, contemporary tattoo parlours using single-use needles.
  • Surface diseases of the skin, hepatitis B, hepatitis C, TB, and HIV are among the pathogens that could theoretically be spread through the use of unsterilized tattoo equipment or contaminated ink. However, no one has been documented to have acquired HIV through a commercial tattooing procedure.
  • Through the bite of an infected mosquito, the chikungunya virus is transmitted to humans. There is no spread of the Chikungunya virus (it cannot be spread directly from person to person).

 

Question 6

Consider the following statements:

  1. Size of the sun at dusk

  2. Colour of the sun at dawn

  3. Moon being visible at dawn

  4. Twinkle of stars in the sky

  5. Polestar being visible in the sky

Which of the above are optical illusions?

(a) 1, 2 and 3

(b) 3, 4 and 5

(c) 1, 2 and 4

(d) 2, 3 and 5

Ans: c

Sub-Theme: Light

Optical Illusion

  • An optical illusion also called a visual illusion that is characterized by visually perceived images that differ from object than reality. The information that is gathered by the eye is processed in the brain that gives a perception that does not tally with a physical measurement of the stimulus source.
  • Statement 1 is correct: Size of the sun at dusk: There are no known objects of a similar size as the sun (or moon) when it is high in the sky. The sun, however, appears even larger when it is low in the sky because we can see distant objects near the horizon that we know to be large.
  • Statement 2 is correct: Colour of the sun at dawn: There are many different colours in light. Red is the colour that moves the farthest out of all of those others. As the sun sets, crimson is the final hue to be seen. Red is the last colour to appear when the other colours begin to fade (red is the fastest).
  • Statement 3 is incorrect: Moon being visible at dawn: The moon is visible somewhere in the sky at dawn during the entire half of the cycle of phases from Full Moon to just before New Moon.
  • Statement 4 is correct: Twinkle of stars in the sky: Light is constantly emitted by stars. Before reaching our sight, this light passes through a number of layers of the earth’s atmosphere. Because different layers of atmosphere have different refractive indices, stars appear to be blinking.
  • Statement 5 is in correct: Polestar being visible in the sky: It is not an optical illusion because Polestars appear in the North or South pole that completely depends on earth rotation.

 

Question 7

The efforts to detect the existence of the Higgs boson particle have become frequent news in the recent past. What is/are the importance/importance of  discovering this particle?

  1. It will enable us to understand why elementary particles have mass.

  2. It will enable us in the near future to develop the technology of transferring matter from one point to another without traversing the physical space between them.

  3. It will enable us to create better fuels for nuclear

Select the correct answer using the codes given below:

(a) 1 only

(b) 2 and 3 only

(c) 1 and 3 only

(d) 1, 2 and 3

Ans: a

Sub-Theme: Scientific innovation and discoveries

Higgs Boson

  • It has long been believed that the Higgs boson holds the key to unlocking the origin of mass puzzle. A phenomenon known as the Higgs field, which is believed to permeate the whole cosmos, is connected to the Higgs boson. According to the theory, as other particles go through this field, they gain mass similarly to how swimmers moving through a pool gain wetness.
  • The Higgs Boson is not the key component that will enable teleportation, but it will aid in our understanding of mass, objects, and time and space travel.
  • The Higgs Boson may, in the near or distant future, assist us in developing an incredible energy source that could open the door to better fuels for our nuclear missions.
  • Statement 1 is correct: The Higgs boson has long been thought the key to resolving the mystery of the origin of mass.
  • Statement 2 is incorrect: The Higgs Boson is not a main item that will open the way for teleportation but it will help to understand about mass, item and travel in time and space.
  • Statement 3 is incorrect: In the near or far future, the Higgs Boson may help us to create an unbelievable energy source which can pave the way for better fuels for our Nuclear missions.

 

Question 8

Fruits stored in a cold chamber exhibit longer storage life because

(a) Exposure to sunlight is prevented

(b) Concentration of carbon dioxide in the environment in increased

(c) Rate of respiration in decreased

(d) There is an increase in humidity

Ans: c

Sub-Theme: Basic and Everyday Science

The Correct answer is Option (c):

  • After being harvested, fruits and vegetables stay metabolically active. Respiration is a manifestation of metabolic The rate of respiration can be decreased effectively by lowering the temperature.
  • Fruits and vegetables can use the energy released during respiration for internal functions. To effectively store your fruit, controlling respiration is crucial.
  • The fruits and vegetables can be preserved in good condition as long as the respiration process can The product will have to survive off of the nutrients it already has because it is no longer connected to the plant’s roots or other sections, so it won’t receive any additional nutrients. The fruits and veggies will therefore start to degrade after everything is done.

 

Question 9

Ball bearings are used in bicycles, cars, , because–

(a) The actual area of contact between the wheel and axle is increased

(b) The effective area of contact between the wheel and axle is increased

(c) The effective area of contact between the wheel and axle is reduced

(d) None of the above statements is correct

Ans: c

Sub-Theme: Basic and Everyday Science

The Correct answer is Option (c): About Ball bearings:

  • Ball bearings are used to reduce friction between the surfaces.
  • A ball bearing is a specific kind of rolling- element bearing that uses balls to keep the bearing races apart. A ball bearing’s main functions are to support radial and axial loads and lessen rotational friction.
  • In order to reduce friction between surfaces and also because the effective area of contact between the wheel and axle is reduced, ball bearings are used in bicycles, vehicle axles, etc.
  • The form of a ball bearing is spherical. These don’t drag each other when they move; instead, they roll. These can be moved with a little force and When compared to static friction and dynamic friction, rolling friction—which is what moves these—is the least forceful.

 

Question 10

Rainbow is produced when sunlight falls on drops of rain. Which of the following physical phenomena are responsible for this?

  1. Dispersion

  2. Refraction

  3. Internal reflection

Select the correct answer using the codes given below:

(a) 1 and 2 only

(b) 2 and 3 only

(c) 1 and 3 only

(d) 1, 2 and 3

Ans: d

Sub-Theme: Basic and Everyday Science

The Correct answer is Option (d): About Rainbow:

  • A rainbow  is  a  meteorological phenomenon which is the result of reflection, refraction and dispersion of light in water droplets resulting in a spectrum of light in the sky.
  • Rainbow takes the form of a multi coloured circular arc. Rainbows due to sunlight always appear in the portion of sky directly opposite the Sun.

 

Question 11

Many transplanted seedlings do not grow because

(a) The new soil does not contain favourable minerals

(b) Most of the root hairs grip the new soil too hard

(c) Most of the root hairs are lost during transplantation

(d) Leaves get damaged during transplantation

Ans: c

Sub-Theme: Transplantation

About Transplanted Seedlings:

  • It typically results from injury to the plant’s roots during transplantation. The majority of the water that is finally delivered to the plant’s top growth is absorbed by tiny roots that are farther from the plant and are covered by even smaller hairs.
  • These delicate roots are typically killed after a transplant either by being chopped to reduce the size of the root ball or by drying out (some accounts say it only takes 3-4 minutes of exposure to air for these to die). It’s referred to as transplant shock.
  • Transplant shock refers to a number of stresses that occur in recently transplanted trees and shrubs. It involves failure of the plant to root well, consequently the plant establishes poorly in the landscape.
  • New transplants frequently experience stress due to a shortage of water because their root systems are not yet well established. Water stressed plants may be more vulnerable to harm from other factors, such as the environment, insects, or

 

Question 12

The known forces of nature can be divided into four classes, viz., gravity, electromagnetism, weak nuclear force and strong nuclear force. With reference  to them, which one of the following statements is not correct?

(a) Gravity is the strongest of the four

(b) Electromagnetism acts only on particles with an electric charge

(c) Weak nuclear force causes radioactivity

(d) Strong nuclear force holds protons and neutrons inside the nucleus of an atom

Ans: a

Sub-Theme: Basic and Everyday Science

The Correct answer is Option (a):

  • Fundamental Forces of Nature:
    • The Fundamental Forces of Nature can be categorised in four categories like
    • Gravitational force, Weak Nuclear force, Electromagnetic force and Strong Nuclear force.
    • The weak and strong forces are influential only over a very short range and dominate only at the level of subatomic particles.
    • Gravity and Electromagnetic force have unlimited range.
  • Gravitational Force – Weakest force; but has unlimited range. (Not part of the standard model). Gravitational Force is not the strongest force rather the weakest
  • The Remaining three forces belong to the standard model of physics.
  • Strong Nuclear Force – Strongest; but has short range.

Strong Nuclear Force > Electromagnetic Force > Weak Nuclear Force > Gravitational Force.

  • Between moving charged particles, the magnetic force operates. This means that regardless of movement, every charged particle emits an electric field. Hence Electromagnetism functions exclusively on particles with an electric charge.
  • The weak nuclear force can transform a neutron into a proton in a process called nuclear decay. Subatomic particles are released at speeds close to the speed of light as a neutrally charged neutron is changed into a positively charged proton by the weak nuclear force.
  • The nuclear fusion reaction that powers the Sun is started by this (weak nuclear) force, which also controls the decay of unstable subatomic particles like mesons.
  • The strong nuclear force that holds protons and neutrons together holds the atom’s nucleus together. Although the strong nuclear force is the strongest of the four fundamental forces, it acts only over very short – typically nuclear – distances. It binds together the protons and neutrons in the nucleus.

 

2012

 

Question 1

What are the reasons for the people’s resistance to the introduction of Bt brinjal in India?

  1. Bt brinjal has been created by inserting a gene from a soil fungus into its genome.

  2. The seeds of Bt brinjal are terminator seeds and therefore, the farmers have to buy the seeds before every season from the seed companies.

  3. There is an apprehension that the consumption of Bt brinjal may have adverse impact on health.

  4. There is some concern that the introduction of Bt brinjal may have adverse effects on biodiversity.

Select the correct answer using the codes given below:

(a) 1, 2 and 3 only

(b) 2 and 3 only

(c) 3 and 4 only

(d) 1, 2, 3 and 4

Ans: c

Sub-Theme: GM Crops

  • Statement 1 is incorrect: A gene called cry1Ac from the soil bacterium Bacillus thuringiensis was inserted into the brinjal to develop Bt Brinjal, a transgenic brinjal.
  • This brinjal has been genetically modified to withstand insects like the Brinjal Fruit and Shoot Borer (Leucinodes orbonalis).
  • Statement 2 is incorrect: Farmers will be free to save and sow as many seeds from Bt Brinjal open-pollinated cultivars as they like. They are not terminator seeds.
  • Statement 3 is correct: A Supreme Court- appointed expert group advised a 10-year moratorium on field trials of all genetically modified (GM) food crops and a complete ban on field trials of transgenics in crops of Indian origins when the case reached that court because consumption of Bt brinjal may have adverse impact on the health of people. Consequently, the government put a freeze on field trial of Bt Brinjal in 2010.
  • Statement 4 is correct: The government’s own task committee on biotechnology had recommended in 2004 that no GM crops be allowed in biodiversity-rich areas since biodiversity is essential for nutrition and sustainability.

 

Question 2  

Other than resistance to pests, what are the prospects for which genetically engineered plants have been created? 

  1. To enable them to withstand the draught.

  2. To increase the nutritive value of the produce.

  3. To enable them to grow and do photosynthesis in spaceships and space stations.

4.To increase their shelf life.

Select the correct answer using the codes given below:

(a) 1 and 2 only

(b) 3 and 4 only

(c) 1, 2 and 4 only

(d) 1, 2, 3 and 4

Ans: c

Sub-Theme: GM Crops

  • Genetically modified (GM) is a technology that involves inserting DNA into the genome of an Organism. The cells are typically cultured in tissue culture after which they transform into plants. The modified DNA will be passed along to the seeds that these plants generate.
  • Statement 1 is correct: Future conflicts may be avoided with the introduction of genetically engineered (GE) drought-tolerant crops that can tolerate longer and more intense droughts. Statement 2 is correct: Nutritional improved GM crops being developed include wheat variants free of gluten, a common trigger for food allergies, and vegetables with increased iron and vitamin A levels to avoid common nutritional deficits in developing countries.
  • Statement 3 is incorrect: Genetically Engineered plants have not been created to enable them to be grown in spacecraft, space stations, and submarines to perform photosynthesis.
  • Statement 4 is correct: Israel has created transgenic, longer-lasting banana plants. They can slow down plant ageing and give fruits like tomatoes an extra week of shelf life.

 

Question 3

Consider the following kinds of organisms:

  1. Bacteria

  2. Fungi

  3. Flowering plants

Some species of which of the above kinds of organisms are employed biopesticides?

(a) 1 only

(b) 2 and 3 only

(c) 1 and 3 only

(d) 1, 2 and 3

Ans: d

Sub-Theme: Biopesticides

Option (d) is correct: All of the above mentioned can be employed as Biopesticides, depending on the properties of specific species. Biopesticides are various biologically active natural substances that exist in nature and have inhibitory effects on crop diseases and insect pests. In general, the creation of biopesticide depends to a large extent on biological resources.

  • Three major classes of biopesticides are:
    • Microbial pesticides consist of a microorganism (e.g., a bacterium, fungus, virus or protozoan) as the active ingredient used to control pests. The microorganism may occur naturally, be dead or alive, or be genetically engineered.
    • Biochemical pesticides are naturally occurring substances, such as plant extracts, fatty acids or pheromones, that control pests using a nontoxic mode of action to the pest.
    • Plant-Incorporated-Protectants (PIPs) are pesticidal substances that plants produce from genetic material that has been added to the plant, such as corn and cotton.
  • All of the above mentioned can be employed as Biopesticides, depending on the properties of specific species.

 

Question 4  

What is the role of ultraviolet (UV) radiation in the water purification systems?

  1. It inactivates/kills the harmful microorganisms in water.

  2. It removes all the undesirable odours from the water.

  3. It quickens the sedimentation of solid particles, removes turbidity and improves the clarity of water.

Which of the statements given above is/are correct?

(a) 1 only

(b) 2 and 3 only

(c) 1 and 3 only

(d) 1, 2 and 3

Ans: a

Sub-Theme: UV Rays

Ultraviolet water purification is the best way for eliminating bacteria from water is ultraviolet water purification.

Application of UV Rays in Purifying the Water:

Statement 1 is correct: A UV water purifier uses germicidal ultraviolet radiation to clean water that is microbiologically hazardous. Living things in the water have their DNA scrambled by the UV wavelength, making it impossible for them to reproduce and infect us. Statement 2 is incorrect: UV disinfection renders living organisms inactive, but it does not clean the water, add chemicals, or get rid of unpleasant tastes and aromas.

Statement 3 is incorrect: For UV purification to be effective, water turbidity (i.e., the quantity of suspended & colloidal materials included in the water to be treated) must be minimal and the water must be clear.

 

Question 5

With reference to ‘stem cells’, frequently in the news, which of the following statements is/are correct?

  1. Stem cells can be derived from mammals only.

  2. Stem cells can be used for screening new drugs.

  3. Stem cells can be used for medical therapies.

Select the correct answer using the codes given below:

(a) 1 and 2 only

(b) 2 and 3 only

(c) 3 only

(d) 1, 2 and 3

Ans: b

Sub-Theme: Stem cell and its applications

  • Stem cells offer hope for the treatment of several disorders for which there is no known cure.
    • They are capable of being pluripotent (a stem cell can grow into a complete organ).
    • Statement 1 is incorrect: Stem cells can be extracted from embryos and body tissues of any species, not just mammals.
    • Statement 2 is correct: Stem cells can be utilised to test new medications and for therapeutic purposes.
    • These stem cells can be extracted from a range of tissue sources, including the brain, lung, heart, muscle, and umbilical cord, and they can be easily grown in vitro.
    • Statement 3 is correct: In stem cell transplants, stem cells either replace diseased or chemo-damaged cells or work with the immune system of the donor to combat certain cancers and blood-related illnesses such leukaemia, lymphoma, neuroblastoma, and multiple myeloma.
    • Adult stem cells or umbilical cord blood are used in these transplants.
  • NOTE: Stem cell and its applications frequently appear in the news. Therefore, it is advisable to thoroughly read the daily newspaper and do some basic research on the particular topic to understand any concepts.

 

Question 6

Which of the following is/are cited by the scientists as evidence/evidence for the continued expansion of the universe? 

  1. Detection of microwaves in space.
  2. Observation of redshirt phenomenon in space.
  3. Movement of asteroids in space.
  4. Occurrence of supernova explosion in space.

Select the correct answer using the codes given below:

(a) 1 and 2

(b) 2 only

(c) 1, 3 and 4

(d) None of the above can be cited as evidence

Ans: a

Sub-Theme: Theories related to space and universe

Expansion of Universe

  • The Big Bang theory states that at some point of time the entire space was contained in a single point of very high density and temperature, from where the Universe has been witnessing expansion in all directions ever since.
  • Redshift and Blueshift describe how light changes with movement of objects in space closer or farther from us. While moving away, the light is shifted to the red end of the spectrum, as its wavelength gets longer. While moving closer, the light moves to the blue end of the spectrum, as its wavelength gets shorter. Galactic Redshift is related to the expanding Universe
  • Cosmic Microwave Background (CMB) is thought to be leftover radiation from the Big Bang, e, the time when the universe began. According to the theory, when the universe was born it underwent rapid inflation and expansion. (The universe is still expanding today, and the expansion rate appears different depending on observation). The CMB represents the heat left over from the Big Bang.
  • According to Hubble’s law, the expansion of the universe is such that the velocity at which the galaxy is moving away from the observer is increasing with time. It means that the universe will become colder increasingly as matter spreads across Space.
  • Statement 1 is correct: Cosmic Microwave Background (CMB) is thought to be leftover radiation from the Big Bang, or the time when the universe began. As the theory goes, when the universe was born it underwent rapid inflation and expansion.
  • Statement 2 is correct: American astronomer Edwin Hubble was the first to describe the redshift phenomenon (galactic redshift) and tie it to an expanding universe (galaxies are drifting apart).
  • Statement 3 is incorrect: Movement of asteroids in space has nothing to do with the concept of expansion of the universe.
  • Statement 4 is incorrect: Occurrence of supernova explosion in space is not related to continued expansion of the Universe.

 

Question 7 

Biomass gasification is considered to be one of the sustainable solutions to the power crisis in India. In this context, which of the following statements is/are correct?

  1. Coconut shells, groundnut shells and rice husk can be used in biomass gasification.

  2. The combustible gases generated consist of hydrogen and carbon dioxide only.

  3. The combustible gases generated from biomass gasification can be used for direct heat generation but not in internal combustion engines.

Select the correct answer using the codes given below:

(a) 1 only

(b) 2 and 3 only

(c) 1 and 3 only

(d) 1, 2 and 3

Ans: a

Sub-Theme: Non-Conventional Energy/ Renewable Energy

  • Statement 1 is correct: Rice husk, groundnut shells, and coconut shells can all be utilised in the biomass gasification process.
  • Statement 2 is incorrect: Biomass gasification involves burning of biomass in a limited supply of air to provide a combustible gas consisting of carbon monoxide, carbon dioxide, hydrogen, methane, water, nitrogen, along with contaminants like small char particles, ash and tars.
  • Statement 3 is incorrect: Combustible syngas is frequently used as a fuel for internal combustion engines or as a step in the synthesis of other compounds. The only combustible gases produced by biomass gasification are carbon dioxide and hydrogen. Consequently, the flammable gases produced by biomass gasification can be used for direct heat generation as well as in internal combustion engines.

 

Question 8 

Graphene has been frequently in the news recently. What is its importance?

  1. It is a two-dimensional material and has good electrical conductivity.

  2. It is one of the thinnest but strongest materials tested so far.

  3. It is entirely made of silicon and has high optical transparency.

  4. It can be used as ‘conducting electrodes’ required for touch screens, LCDs and organic LEDs.

Which of the statements given above are correct?

(a) 1 and 2 only

(b) 3 and 4 only

(c) 1, 2 and 4 only

(d) 1, 2, 3 and 4

Ans: c

Sub-Theme: Basic and Everyday Science

  • Statement 1 is correct: Graphene, a two- dimensional, atomic-scale, honey-comb lattice is an allotrope of carbon in which one atom forms each vertex. It is the basic structural unit of other allotropes, including graphite, charcoal, carbon nanotubes and fullerenes.
  • Statement 2 is correct: It is about 100 times stronger than the strongest steel with a hypothetical thickness of 3.35. A which is equal to the thickness of a graphene sheet. Carbon atoms are linked together in a single layer to form the hexagonal structure that makes up graphene. Graphene is so much thinner than paper (a million times thinner), that it is actually regarded as two dimensional.
  • Statement 3 is incorrect: Graphene is a single layer (monolayer) of carbon atoms, tightly bound in a hexagonal honeycomb lattice. It is an allotrope of carbon in the form of a plane of sp2-bonded atoms with a molecular bond length of 0.142 nanometres.
  • Statement 4 is correct: Graphene is often produced as a powder and as a dispersion in a polymer matrix. This dispersion is supposedly suitable for advanced composites, paints and coatings, lubricants, oils and functional fluids, capacitors and batteries, thermal management applications, display materials, etc.

 

Question 9

Lead, ingested or inhaled, is a health hazard. After the addition of lead to petrol has been banned, what still are the sources of lead poisoning?

  1. Smelting units

  2. Pens and pencils

  3. Paints

  4. Hair oils and cosmetics

Select the correct answer using the codes given below:

(a) 1, 2 and 3 only

(b) 1 and 3 only

(c) 2 and 4 only

(d) 1, 2, 3 and 4

Ans: b

Sub-Theme: Environmental pollution

The Correct answer is Option (b): About Lead:

  • The crust of the Earth contains the naturally occurring hazardous element, lead. Due to its widespread use, there have been serious public health issues, environmental damage, and human exposure in many different parts of the world.
  • Mining, smelting, manufacturing, recycling, and, in some nations, the continuous use of leaded paint and aviation
  • fuel are important sources of environmental contamination. There is no relationship between lead poisoning and Pen and Pencil, Hair oil and Cosmetics .
  • The production of lead-acid batteries for automobiles accounts for more than three quarters of all lead usage worldwide.
  • Some sources of Lead Poisoning:
    • Lead-Based Paint
    • Contaminated Soil
    • Children’s Toys and Jewellery
    • Drinking Water delivered through lead pipes
    • Workplace and Hobby Hazards (mining, smelting)
    • Lead Glazed Ceramic Ware, Pottery and Leaded Crystal
    • Imported Candy
    • Mini Blinds

 

Question 10

Consider the following crops of India:

  1. Cowpea

  2. Green gram

  3. Pigeon pea

Which of the above is/are used as pulse, fodder and green manure?

(a) 1 and 2 only

(b) 2 only

(c) 1 and 3 only

(d) 1, 2 and 3

Ans: a

Sub-Theme: Basic and Everyday Science

The Correct answer is Option (a):

  • Cowpea and Green gram are known as pulses because they are a good source of protein and are widely used as a staple food in many parts of the world. However, they also have other uses, such as being used as fodder and green manure.
  • As fodder, Cowpea and Green gram are highly nutritious and can provide a good source of protein for They are also easy to grow and can be grown in a variety of conditions, making them an ideal choice for farmers looking to provide their animals with a healthy and sustainable food source.
  • As green manure, Cowpea and Green gram are known for their ability to fix nitrogen in the soil, which can help to improve soil fertility and increase crop yields. They are often used in crop rotations, where they are grown for a season and then plowed back into the soil to improve its nutrient
  • Pigeon pea or Toor Dal is a traditionally Kharif crop sown in June-July with the onset of Monsoon in various agro-climatic zones of India.
    • Pigeon Pea belongs to the widespread family of pulses. In India Pigeon Pea is more popular as Arhar or red gram In India, split pigeon peas (toor dal) are one of the most popular pulses.
    • Being an important source of protein in a mostly vegetarian diet. In regions where it grows, fresh young pods are eaten as a vegetable.
    • The split dried seeds are used as a lentil, in dishes such as sambar (lentil soup).

 

Question 11 

To meet its rapidly growing energy demand, some opine that India should pursue research and development on thorium as the future fuel of nuclear  energy. In this context, what advantage does thorium hold over uranium?

  1. Thorium is far more abundant in nature than

  2. On the basis of per unit mass of mined mineral, thorium can generate more energy compared to natural uranium.

3.Thorium produces less harmful waste compared to uranium.

Which of the statements given above is/are correct?

(a) 1 only

(b) 2 and 3 only

(c) 1 and 3

(d) 1, 2 and 3

Ans: d

Sub-Theme: Nuclear Energy

  • Statement 1 is correct: Thorium composes 0.0006% of the earth’s crust whereas Uranium composes 0.00018% of the earth’s crust, where a substantial amount of Uranium is found in dissolved seawater.
  • Statement 2 is correct: It is predicted that uranium-233 produced from thorium can produce 8 times more energy per unit mass than (natural) uranium.
  • Statement 3 is correct: There is up to two orders of magnitude less nuclear waste in the liquid fluoride thorium reactor, eliminating the need for large-scale and long-term storage for the waste.

 

Question 12

Which one of the following sets of elements was primarily responsible for the origin of life on the Earth? 

(a) Hydrogen, Oxygen, Sodium

(b) Carbon, Hydrogen, Nitrogen

(c) Oxygen, Calcium, Phosphorus

(d) Carbon, Hydrogen, Potassium

Ans: b

Sub-Theme: Basic and Everyday Science

The Correct answer is Option (d):

  • The correct answer is Carbon, Hydrogen, Nitrogen. Carbon and Hydrogen become hydrocarbons and further hydrocarbon chains in the cell membranes are a fundamental component of life.

 

Question 13

Consider the following statements: If there were no phenomenon of capillarity 

  1. It would be difficult to use a kerosene lamp.

  2. One would not be able to use a straw to consume a soft drink.

  3. The blotting paper would fail to function.

  4. The big trees that we see around would not have grown on the Earth.

Which of the statements given above are correct?

(a) 1, 2 and 3 only

(b) 1, 3 and 4 only

(c) 2 and 4 only

(d) 1, 2, 3 and 4

Ans: b

Sub-Theme: Basic and Everyday Science

  • Statement 1 is correct: The oil in the wick of a lamp rises due to the capillary action of threads in the wick.
  • Statement 2 is incorrect: No capillary action in using a straw to consume the soft drink as we applied additional pressure from the mouth.
  • Statement 3 is correct: A blotting paper soaks ink by the capillary action of the pores in the blotting paper.
  • Statement 4 is correct: The root-hairs of plants draw water from the soil through capillary action.

 

2011

 

Question 1 

At present, scientists can determine the arrangement or relative positions of genes of DNA sequences on a chromosome. How does this knowledge benefit us?

  1. It is possible to know the pedigree of

  2. It is possible to understand the causes of all human diseases.

  3. It is possible to develop disease-resistant animal

Which of the statements given above is/are correct?

(a) 1 and 2 only

(b) 2 only

(c) 1 and 3 only

(d) 1, 2 and 3

Ans: c

Sub-Theme: DNA sequences

  • DNA Sequencing means determining the order of the four chemical building blocks – called “bases” – that make up the DNA molecule. This sequence informs scientists the kind of genetic information that is carried in a particular DNA segment.
  • Statement 1 is correct: A thorough pedigree analysis using pedigree files received via DNA sequencing from the relevant breed organisations.
  • Statement 2 is incorrect: Sequencing enables medical professionals to identify whether a gene or the region that controls a gene includes alterations, known as variations or mutations, that are connected to an illness. But, through arrangement of genes of DNA sequences on a chromosome, it is not possible to understand the causes of all human diseases.
  • Statement 3 is correct: Researchers have even been able to identify specific faults in genes that are responsible for or contribute to disease. Hence, Breeds of animals that are resistant to disease can be created.

 

Question 2

Regular intake of fresh fruits and vegetables is recommended in the diet since they are a good source of antioxidants. How do antioxidants help a person maintain health and promote longevity?

(a) They activate the enzymes necessary for vitamin synthesis in the body and help prevent vitamin deficiency

(b) They prevent excessive oxidation of carbohydrates, fats and proteins in the body and help avoid unnecessary wastage of energy

(c) They neutralize the free radicals produced in the body during metabolism

(d) They activate certain genes in the cells of the body and help delay the ageing process

Ans: c

Sub-Theme: Component of food/balanced diet

Option (c) is correct: The purpose of antioxidants is to counteract the harmful effects of free radicals in our body, thereby

promoting better health. By providing an electron to a roaming free radical, antioxidants have the ability to stabilize it, thus mitigating its potential to cause damage.

About Antioxidant:

  • Any substance that can inhibit the formation of unstable molecules known as free radicals, which harm DNA, cell membranes, and other components of cells, is referred to as a “antioxidant.”
  • Free radicals grab electrons from other molecules in order to complete their electron complement, which causes those molecules to suffer damage.
  • Free radicals are countered by antioxidants by sacrificing some of their own electrons. They serve as a natural “off” switch for the free radicals by making this sacrifice. Antioxidants neutralize the free radicals produced in the body during
  • For example some vitamins and minerals — including vitamins C and E and the minerals copper, zinc, and selenium — serve as antioxidants, in addition to other vital roles.
    • Antioxidants can also be found in food, especially in fruits, vegetables, and other plant-based, whole foods.

 

Question 3

What is the difference between Bluetooth and Wi-Fi devices?

(a) Bluetooth uses 2.4 GHz radio frequency band, whereas Wi-Fi can use 4 GHz or 5 GHz frequency band

(b) Bluetooth is used for Wireless Local Area Networks (WLAN) only, whereas Wi-Fi is used for Wireless Wide Area Networks (WWAN) only

(c) When information is transmitted between two devices using Bluetooth technology, the devices have to be in the line of sight of each other, but when Wi-Fi technology is used the devices need not be in the line of sight of each other

(d) The statements (a) and (b) given above are correct in this context

Ans: a

Sub-Theme: Communication Technology

Bluetooth WiFi
It is a wireless technology that is used for exchanging data between mobile devices over short distances using radio waves. It is a wireless networking technology that uses radio waves to provide wireless

high-speed internet and network connections.

Requires Bluetooth adaptor Requires wireless adaptor and router
Frequency 2.4Ghz Frequency 2.4-5Ghz
Consumes Low power Consumes high power
Are Less secure Are More secure

 

No need for devices to be in line of sight; they just need to be within bluetooth’s range. No need for devices to be in line of sight, they just need to be within range of WiFi
Uses WPAN- wireless personal area network Uses WLAN- wireless local area network
Coverage 10 meters Coverage 32 meters
Needs low bandwidth Needs high bandwidth

 

Question 4   

What is a “Virtual Private Network”?

(a) It is a private compute network of an organization where he remote users can transmit encrypted information through the server of the

(b) It is a computer network across a public internet that provides users access to their organization’s network while maintaining the security of the information transmitted.

(c) It is a computer network in which users can access a shared pool of computing resources through a service provider

(d) None of the statements (a), (b) and (c) given above is a correct description of Virtual Private

Ans: b

Sub-Theme: Communication Technology

The Correct answer is Option (b):

  • VPN: An encrypted connection between a device and a network via the Internet is known as a virtual private network, or VPN. Secure transmission of sensitive data is aided by the encrypted It makes it impossible for unauthorised parties to eavesdrop on the traffic and enables remote work for the user.
  • By creating secure connections over the Internet, a VPN expands a company network. Traffic stays private while travelling because it is encrypted between the device and the
  • Even smartphones and tablets can connect through a VPN.

 

Question 5  

An artificial satellite orbiting around the Earth does not fall down. This is so because the attraction of Earth

(a) Does not exist at such distance

(b) Is neutralized by the attraction of the moon

(c) Provides the necessary speed for its steady motion

(d) Provides the necessary acceleration for its motion

Ans: d

Sub-Theme: Basic knowledge on space/ Universe/Gravitation

The Correct answer id Option (d):

  • Artificial satellites are launched into orbit for a variety of purposes, including communications, navigation, Earth imaging, weather observation, and scientific study.
  • In satellites, humans typically feel weightless. We experience a sense of weightlessness because the net force acting on our body is practically zero.This is due to the satellite’s centrifugal force, which cancels out the size of the earth’s gravitational pull.
  • On the other hand, the earth’s gravitational pull is required for the satellite to continuously change its direction, giving acceleration for the satellite’s direction- changing process.
  • When artificial satellites are sent into outer space, a huge velocity is applied to them to escape the earth’s gravitation At that point, they develop their own velocity and acceleration, with respect to the other forces acting on them to move in a path in accordance to earth’s revolution around the sun.

 

Question 6  

Which is the difference between asteroids and comets?

  1. Asteroids are small rocky planetoids, while comets are formed of frozen gases held together by rocky and metallic material.

  2. Asteroids are found mostly between the orbits of Jupiter and Mars, while comets are found mostly between Venus and Mercury.

  3. Comets show a perceptible glowing tail, while asteroids do not.

Which of the statements given above is/are correct?

(a) 1 and 2 only

(b) 1 and 3 only

(c) 3 only

(d) 1, 2 and 3

Ans: b

Sub-Theme: Basic knowledge on space/Universe

  • Asteroids are rocky, airless remnants left over from the early formation of our solar system about 6 billion years ago. They are sometimes called minor planets.
  • Comets are frozen leftovers from the formation of the solar system composed of dust, rock, and ices. Their size ranges from a few miles to tens of miles wide, and as they orbit closer to the Sun, they heat up and spew gases and dust into a glowing head that can be larger than a planet.
  • This material forms a tail that stretches millions of miles.
  • The main asteroid belt, which extends between Mars and Jupiter, is home to the majority of this old space debris. Asteroids range in size from the largest, Vesta, which has a diameter of roughly 329 miles (530 kilometres), to bodies that are about 33 feet (10 metres) wide. The cumulative mass of all asteroids is less than that of the moon of Earth.
  • Comets are solar system-orbiting snowballs made of frozen gases, rock, and dust.
  • They enlarge to the size of a sizable town when frozen.
  • A comet heats up and ejects dust and gases into a huge, blazing head that is larger than most planets when its orbit puts it near to the Sun.
  • Millions of kilometres away from the Sun, the dust and gas combine to form a The Kuiper Belt and much further Oort Cloud are presumably home to billions of comets that orbit our Sun.
  • Comets are not limited to the belt between Mercury and Venus.
  • Statement 1 is correct: Asteroids, sometimes called minor planets, are rocky, airless remnants left over from the early formation of our solar system about 4.6 billion years ago. Comets are frozen leftovers from the formation of the solar system composed of dust, rock, and ices.
  • Statement 2 is incorrect and 3 is correct: When a comet’s orbit brings it close to the Sun, it heats up and spews dust and gases into a giant glowing head larger than most planets. Also, comets are not restricted to the belt between Mercury and Venus.

 

Question 7  

Satellites used for telecommunication relay are kept in a geostationary orbit. A satellite is said to be in such an orbit when:

  1. The orbit is geosynchronous.

  2. The orbit is circular.

  3. The orbit lies in the plane of the Earth’s

  4. The orbit is at an altitude of 22,236

Select the correct answer using the codes given below:

(a) 1, 2 and 3 only

(b) 1, 3 and 4 only

(c) 2 and 4 only

(d) 1, 2, 3 and 4

Ans: a

Sub-Theme: Space technology

Geostationary Orbit

  • Geostationary Orbit is a type of geosynchronous orbit.
  • The earth rotates in the same direction that satellites do. It lasts for roughly 24 hours and holds its place in relation to the earth.
  • Low earth orbit, medium earth orbit, geosynchronous orbit, geostationary orbit, etc. are several categories for circular orbits. Thus, communications satellites are positioned in elliptical orbits.
  • Geostationary orbits are in the same plane as the equator as opposed to geosynchronous satellites, which can have any inclination.
  • Geostationary orbit lies at an altitude of about 36000 km.
  • Statement 1 is correct: Geostationary Orbit is one type of geosynchronous orbit only Statement 2 is correct: Circular orbits are classified into low earth orbit, medium earth orbit, geosynchronous orbit, geostationary orbit etc. Hence telecommunication satellites are placed in circular orbits.
  • Statement 3 is correct: While geosynchronous satellites can have any inclination, geostationary orbits lie on the same plane as the equator.
  • Statement 4 is incorrect: Geostationary orbit is at an altitude of around 36000 km.

 

Question 8   

The surface of a lake is frozen in severe winter, but the water at its bottom is still liquid. What is the reason?

(a) Ice is a bad conductor of heat

(b) Since the surface of the lake is at the same temperature as the air, no heat is lost

(c) The density of water is maximum at 4°C

(d) None of the statements (a), (b) and (c) given above is correct

Ans: c

Sub-Theme: Basic and Everyday Science

The Correct answer is Option (c): About Water:

  • Water attains maximum density at
  • The cold water at the top sinks to the bottom when water at a temperature higher than this is cooled from above, allowing warmer water to be cooled.
  • Nevertheless, this alters as the temperature reaches 4°C. The cold water condenses on the surface and selectively cools until it turns to ice. Because of how well this ice retains heat, it “protects” the warmer water beneath it from freezing.
  • But depending on the temperature of the atmosphere, the thickness of the ice layer would change.

 

Question 9   

Microbial fuel cells are considered a source of sustainable energy. Why?

  1. They use living organisms as catalysts to generate electricity from certain substrates.

  2. They use a variety of inorganic materials as substrates.

  3. They can be installed in wastewater treatment plants to cleanse water and produce electricity.

Which of the statements given above is/are correct?

(a) 1 only

(b) 2 and 3 only

(c) 1 and 3 only

(d) 1, 2 and 3

Ans: c

Sub-Theme: Non-Conventional Energy/ Renewable Energy/Sustainable Energy

The Correct answer is Option (c): About Microbial fuel cells:

microbial fuel cells

  • A novel bio electrochemical technique called microbial fuel cells (MFCs) proposes to generate power using electrons from biological reactions that bacteria catalase.
  • It is anticipated that the energy produced by MFCs will be sufficient to partially meet the energy requirement in urban waste water treatment plants (WWTPs).
  • A Microbial fuel cell (MFC) contains an anode and a cathode divided by a cation exchange Microorganisms oxidise organic compounds (not inorganic) in the anode chamber, producing electrons and protons (reaction of anodes).

 

Question 10

The function of heavy water in a nuclear reactor is to

(a) Slow down the speed of neutrons

(b) Increase the speed of neutrons

(c) Cool down the reactor

(d) Stop the nuclear reaction

Ans: a

Sub-Theme: Nuclear Energy

The Correct answer is Option (a): About Heavy Water:

  • Moderator is used in a Nuclear reactor for slowing down the neutrons produced during the fission reaction so that the chain reaction can be sustained.
  • Due to its high moderating ratio and low neutron absorption cross section, heavy water makes a good moderator. Deuterium oxide, popularly known as heavy water (D2O), is a hydrogen isotope with a mass double that of regular hydrogen and It is a component of water.
  • In nuclear power reactors, heavy water is utilised as a coolant and a neutron moderator.
    •   Components of a nuclear reactor:

    0 Fuel – which is generally Uranium

    0 Moderator – It is used to slow down the neutrons released from fission to create more fission. Generally, water, heavy water or graphite is used for this purpose.

    0 Control rods or blades – They are basically made with neutron absorbing material such as cadmium, hafnium, boron and are inserted from the core to control the rate of reaction.

    0 Coolant- It is a fluid circulating through the core so as to transfer the heat from it.

    Pressure vessel or pressure tubes

    – It is usually a robust steel vessel containing the reactor core and moderator or coolant but it may be a series of tubes holding the fuel and conveying the coolant through the surrounding moderator.

    0 Steam generator – It is the part of the cooling system of pressurized water reactors where the high pressure primary coolant brings heat from the reactor.

 

Question 11

A company marketing food products advertises that its items do not contain trans fats. What does this campaign signify to the customers?

  1. The food products are not made out of hydrogenated oils.

  2. The food products are not made out of animal fats/oils.

  3. The oils used are not likely to damage the cardiovascular health of the consumers.

Which of the statements given above is/are correct?

(a) 1 only

(b) 2 and 3 only

(c) 1 and 3 only

(d) 1, 2 and 3

Ans: c

Sub-Theme: Food product regulations

  •  Statement 1 and 3 are correct: If items do not contain trans-fats, it means the food products are not made out of hydrogenated oils. If items do not contain trans-fats, it means oils used are not likely to damage the cardiovascular health of the consumers
  • Statement 2 is incorrect: The food products consisting Trans- fats can be made using animal fats/oils. E.g Meat and Dairy products.

 

Question 12

What is the difference between a CFL and an LED lamp?

  1. To produce light, a CFL uses mercury vapour and phosphor while an LED lamp uses semiconductor material.

  2. The average lifespan of a CFL is much longer than that of an LED lamp

  3. A CFL is less energy-efficient as compared to an LED lamp.

Which of the statements given above is/are correct?

(a) 1 only

(b) 2 and 3 only

(c) 1 and 3 only

(d) 1, 2 and 3

Ans: c

Sub-Theme: Modern Energy efficient Technology

Statement 1 is correct: An LED is a type of semiconductor light source with two leads. It is a p-n junction diode, which when turned on emits light. A fluorescent lamp is a low- pressure mercury-vapour gas-discharge light.

Statement 2 is incorrect and Statement 3 is correct: LED bulbs are much more efficient than CFL bulbs in terms of power consumption and life years.

 

Question 13

 Aspartame is an artificial sweetener sold in the market. It consists of amino acids and provides calories like other amino acids. Yet, it is used as a low- calorie sweetening agent in food items. What is the basis of this use?

(a) Aspartame is as sweet as table sugar, but unlike table sugar, it is not readily oxidised in human body due to lack of requisite enzymes

(b) When aspartame is used in food processing, the sweet taste remains, but it becomes resistant to

(c) Aspartame is as sweet as sugar, but after ingestion into the body, it is converted into metabolites that yield no calories

(d) Aspartame is several times sweeter than table sugar, hence food items made with small quantities of aspartame yield fewer calories on

Ans: (d)

Sub-Theme: Healthy food/nutrition/Artificial Sweetener.

The Correct answer is Option (d):

  • Aspartame is a non-saccharide artificial sweetener 200 times sweeter than sucrose, and is commonly used as a substitute of sugar in foods and beverages.
  • The commercial names for it are NutraSweet, Equal, and Canderel. It is a methyl ester of the aspartic acid/phenylalanine dipeptide.
  • Because aspartame is much sweeter than table sugar, foods produced with it produce fewer calories when they are oxidised.

 

Question 14  

Recently, “oilzapper” was in the What is it?

(a) It is an eco-friendly technology for the remediation of oily sludge and oil spills

(b) It is the latest technology developed for under-sea oil exploration

(c) It is a genetically engineered high biofuel-yielding maize variety

(d) It is the latest technology to control the accidentally caused flames from oil wells.

Ans: a

Sub-Theme: Eco-friendly technology

    • Oilzapper is an eco-friendly technology for oil sludges and oil spills remediation. Oilzapping is the new technique using bacteria to get rid of oil spills. It is a bio- remediation technique that involves the use of oil zapping bacteria.
    • In essence, Oilzapper is a concoction of five distinct bacterial strains that have been immobilised and combined with a carrier substance (powdered corn cob).
    • The Oilzapper consumes the hydrocarbon molecules found in crude oil and oily sludge, a dangerous hydrocarbon waste product produced by oil refineries, and transforms them into harmless CO2 and water. This is exactly what its witty name implies.

 

Question 15   

A married couple adopted a male child. A few years later, twin boys were born to The blood group of the couple is AB positive and O negative. The blood group of the three sons is A positive, B positive, and O positive. The blood group of the adopted son is

(a) O positive

(b) A positive

(c) B positive

(d) Cannot be determined on the basis of the given data

Ans: a

Sub-Theme: Blood group

The Correct answer is Option (a):

  • The blood group of the couple is AB Positive and O So their biological sons will have either blood group A or B. Thus we can infer that the blood group of the adopted son will be O.

126. (a) Sub-Theme: Blood group The Correct answer is Option (a): • The blood group of the couple is AB Positive and O Negative. So their biological sons will have either blood group A or B. Thus we can infer that the blood group of the adopted son will be O.

 

Question 16

A new optical disc format known as the Blu-ray Disc (BD) is becoming In what way is it different from the traditional DVD?

  1. DVD supports standard Definition video while BD supports High definition Video.

  2. Compared to a DVD, the BD format has several times more storage capacity.

  3. Thickness of BD is 4 mm while that of DVD is 1.2 mm.

Which of the statements given above is/are correct?

(a) 1 only

(b) 1 and 2 only

(c) 2 and 3 only

(d) 1, 2 and 3

Ans: b      

Sub-Theme: Digital Technology

  • Statement 1 is correct: DVD stands for “Digital Versatile Disk” or “Digital Video Disc”. It is a type of digital optical disc storage that can hold any kind of digital data but is frequently used to store pictures, audio, and video. Blu-ray is a digital disc storage format similar to CD and DVD, that is designed to record and play a large amount of data with high-definition quality.
  • Statement 2 is correct: Blu-ray Disc media include these features: 50GB capacity – Each disc can hold more than 10 standard DVDs.
  • Statement 3 is incorrect: The size of the Blu- ray disc is 120 millimetres in diameter with 1.2 millimetres thickness, which is the same as the size of a CD and DVD.

 

Question 17

When the bark of a tree is removed in a circular fashion all around near its base, it gradually dries up and dies because 

(a) Water form soil cannot rise to aerial parts

(b) Roots are starved of energy

(c) Tree is infected by soil microbes

(d) Roots do not receive oxygen for respiration

Ans: (b)         

Sub-Theme: Basic and Everyday Science

The Correct answer is Option (b):

  • The tissue just beneath the bark called the phloem layer is in charge of moving food produced by photosynthesis from the leaves to the roots.
  • Without this nourishment, the roots eventually perish and stop supplying the leaves with water and nutrients. Later, the leaves wither.
  • Because its roots are starved of vitality, a tree that has had its bark removed in a circular pattern all around near its base gradually dries up and dies.

 

Question 18

A layer in the Earth’s atmosphere called Ionosphere facilitates radio communication. Why?

  1. The presence of ozone causes the reflection of radio waves to Earth.

  2. Radio waves have a very long wavelength.

Which of the statements given above is/are correct?

(a) 1 only

(b) 2 only

(c) Both 1 and 2

(d) Neither 1 nor 2

Ans: d

Sub-Theme: Basic and Everyday Science

  • Statement 1 is incorrect: It contains ions and not Ozone is present in the Stratosphere and not in the Ionosphere.
  • Statement 2 is incorrect: They have frequencies from 300 GHz to as low as 3 kHz and corresponding wavelengths from 1 millimetre to 100 kilometres.

 

Question 19

A genetically engineered form of brinjal, known as the Bt-brinjal, has been The objective of this is 

(a) To make it pest-resistant

(b) To improve its taste and nutritive qualities

(c) To make it drought-resistant

(d) To make its shelf-life longer

Ans: a

Sub-Theme: GM Crops

The Correct answer is Option (a):

  • Bt Brinjal is a genetically modified (GM) crop created by inserting Cry1Ac gene from the soil bacterium Bacillus thuringiensis into Brinjal.
  • Brinjal plants now have resistance to lepidopteran pests such the Brinjal Fruit and Shoot Borer (Leucinodes orbonalis) and Fruit Borer as a result of the gene’s insertion (Helicoverpa armigera).

 Final Result – CIVIL SERVICES EXAMINATION, 2023.   Udaan-Prelims Wallah ( Static ) booklets 2024 released both in english and hindi : Download from Here!     Download UPSC Mains 2023 Question Papers PDF  Free Initiative links -1) Download Prahaar 3.0 for Mains Current Affairs PDF both in English and Hindi 2) Daily Main Answer Writing  , 3) Daily Current Affairs , Editorial Analysis and quiz ,  4) PDF Downloads  UPSC Prelims 2023 Trend Analysis cut-off and answer key

THE MOST
LEARNING PLATFORM

Learn From India's Best Faculty

      

 Final Result – CIVIL SERVICES EXAMINATION, 2023.   Udaan-Prelims Wallah ( Static ) booklets 2024 released both in english and hindi : Download from Here!     Download UPSC Mains 2023 Question Papers PDF  Free Initiative links -1) Download Prahaar 3.0 for Mains Current Affairs PDF both in English and Hindi 2) Daily Main Answer Writing  , 3) Daily Current Affairs , Editorial Analysis and quiz ,  4) PDF Downloads  UPSC Prelims 2023 Trend Analysis cut-off and answer key

Quick Revise Now !
AVAILABLE FOR DOWNLOAD SOON
UDAAN PRELIMS WALLAH
Comprehensive coverage with a concise format
Integration of PYQ within the booklet
Designed as per recent trends of Prelims questions
हिंदी में भी उपलब्ध
Quick Revise Now !
UDAAN PRELIMS WALLAH
Comprehensive coverage with a concise format
Integration of PYQ within the booklet
Designed as per recent trends of Prelims questions
हिंदी में भी उपलब्ध

<div class="new-fform">







    </div>

    Subscribe our Newsletter
    Sign up now for our exclusive newsletter and be the first to know about our latest Initiatives, Quality Content, and much more.
    *Promise! We won't spam you.
    Yes! I want to Subscribe.